[HN Gopher] Why are nuclear power construction costs so high?
       ___________________________________________________________________
        
       Why are nuclear power construction costs so high?
        
       Author : spenrose
       Score  : 286 points
       Date   : 2022-06-09 15:22 UTC (7 hours ago)
        
 (HTM) web link (constructionphysics.substack.com)
 (TXT) w3m dump (constructionphysics.substack.com)
        
       | teslaberri wrote:
       | corruption is why, it has become a legal part of doing business.
       | that simple.
        
       | Georgelemental wrote:
       | TL;DR rising labor costs and regulation
        
         | bryanlarsen wrote:
         | Note that the regulatory costs are only responsible for much of
         | the increases of the 60s and 70s. They're the boogey man, but
         | the increases of the last forty years can't be blamed on them.
         | 
         | It's a general problem: HSR and subway stations have seen
         | similar increases.
        
           | NaNDude wrote:
           | may be the increases of the last forty years have'nt been
           | studied as thoses of the 60s to 70s, but regulatory costs are
           | still a reality, the french EPR for exemple got a 20billion
           | increase in cost during the construction for changes in
           | security standards.
        
           | Robotbeat wrote:
           | That's also due to regulatory costs. Regulations now require
           | "citizen voice" for large projects, whether infrastructure or
           | just an apartment building. So NIMBYism is able to slow
           | construction (which translates directly to increased costs),
           | require additional measures or features. And NIMBY lawsuits
           | after regulatory approval is given can further increase costs
           | and schedule, even if the court rules in favor of the
           | project. And then because of all these regulatory costs, the
           | experience isn't gained, so learning doesn't occur, and if it
           | does, it occurs only for a few firms. Additionally, large
           | state sponsored projects often are treated as jobs programs,
           | etc.
           | 
           | Another issue is that the difficulty of complying with the
           | regulations is intentional. The paperwork is difficult as an
           | intentional sort of time-tax on building anything new. We
           | could actually automate and streamline everything to be
           | approved immediately (while following the letter) without
           | large paperwork costs, but that's not actually what those who
           | push for the regulations actually want. They WANT it to be
           | hard.
           | 
           | It's not just a boogeyman. It's the real reason. Regulations
           | are responsible for most of the cost of nuclear power.
        
             | Ericson2314 wrote:
             | I don't think it's useful to conflate construction code
             | style regulation and NIMBYism.
             | 
             | There can be excess of the former, but ultimately rules-
             | based regulation isn't the worst. NIMBYism and other
             | discretionary review adds more much delays and uncertainty
             | and _that_ is the Achilles heel.
             | 
             | Ultimately we need build out the literal and metaphorical
             | supply chains, i.e. do the same thing over and over and
             | over again. Economies of scale are real, and so are
             | diseconomies of discale, and the latter is the NIMBY's
             | greatest weapon for collective action.
             | 
             | Are Stadtbahns the SMRs of transit?
        
             | philipkglass wrote:
             | _It's not just a boogeyman. It's the real reason.
             | Regulations are responsible for most of the cost of nuclear
             | power._
             | 
             | In a counterfactual world with fewer regulations, I can
             | believe that construction would be cheaper. But regulations
             | don't explain why new projects have drastic cost and
             | schedule overruns. The United States started building new
             | AP1000 reactors in Georgia and South Carolina in 2013 [1]
             | [2]. There were no regulatory changes/increases after 2013.
             | But the projects went drastically over the budget and
             | schedule numbers that they had in 2013.
             | 
             | The South Carolina project was ultimately canceled and the
             | SCANA CEO ended up in federal prison:
             | https://www.powermag.com/former-scana-ceo-will-land-in-
             | priso...
             | 
             |  _Following the project 's demise, an "exhaustive and
             | multi-year joint investigation" was conducted by the U.S.
             | Attorney's Office, the Federal Bureau of Investigation, the
             | U.S. Securities and Exchange Commission, the South Carolina
             | Attorney General's Office, and the South Carolina Law
             | Enforcement Division. Marsh's sentencing is the result of
             | that investigation.
             | 
             | "Kevin Marsh deceived regulators and customers to
             | financially benefit SCANA," Susan Ferensic, special agent
             | in charge of the FBI Columbia Field Office, said in a
             | statement. "Unfortunately, Marsh's and other executive's
             | actions resulted in South Carolinians bearing the financial
             | brunt of the failed Summer Nuclear Station."
             | 
             | "Due to this fraud, an $11 billion nuclear ghost town, paid
             | for by SCANA investors and customers, now sits vacant in
             | Jenkinsville, S.C.," DeHart said._
             | 
             | The Georgia project is still in progress.
             | 
             | https://apnews.com/article/business-environment-united-
             | state...
             | 
             | "Georgia nuclear plant's cost now forecast to top $30
             | billion"
             | 
             |  _A nuclear power plant being built in Georgia is now
             | projected to cost its owners more than $30 billion.
             | 
             | A financial report from one of the owners on Friday clearly
             | pushed the cost of Plant Vogtle near Augusta past that
             | milestone, bringing its total cost to $30.34 billion._
             | 
             | ...
             | 
             |  _When approved in 2012, the third and fourth reactors were
             | estimated to cost $14 billion, with the first electricity
             | being generated in 2016. Now the third reactor is set to
             | begin operation in March 2023, and the fourth reactor is
             | set to begin operation in December 2023._
             | 
             | It's reasonable to say that a pacemaker costs more to
             | develop than an MP3 player because medical devices are
             | heavily regulated by the FDA. But it's not reasonable to
             | say that a project to develop a new pacemaker is 100% over
             | budget and 7 years late because of FDA regulations if the
             | FDA regulations didn't change in the mean while. In this
             | case, the regulations didn't change. So earlier estimates
             | were due to fraud or incompetence (either execution-
             | incompetence or planning-incompetence). I tend to blame
             | incompetence -- after all, _many_ megaprojects end up
             | horribly late and over budget, not just nuclear ones -- but
             | in the case of South Carolina 's VC Summer there was
             | outright fraud too.
             | 
             | [1] https://en.wikipedia.org/wiki/Vogtle_Electric_Generatin
             | g_Pla...
             | 
             | [2] https://en.wikipedia.org/wiki/Virgil_C._Summer_Nuclear_
             | Gener...
        
             | jandrese wrote:
             | Is this due to "citizen voice" or is it just that all of
             | the land is now someone's back yard? A century ago most of
             | the land around cities was forest, plains, or sometimes
             | farm. Building out rail, transmission lines, or pipelines
             | was relatively easy because barely anybody lived near where
             | you were building.
             | 
             | Today there are people scattered all around and they will
             | absolutely complain when you start building something near
             | the property they bought specifically to be away from other
             | people.
        
             | bumby wrote:
             | You're not wrong, but I'm not sure what's a better
             | alternative.
             | 
             | Think of it in a different scope: government contracts are
             | also expensive because the have to be open to competition.
             | A lot of the red tape could be reduced with no-bid
             | contracts, but people understand the corruption risk
             | tradeoff is generally not worth it.
             | 
             | In your example, it seems like the NIMBYism is the root
             | cause, not the process by which NIMBYism is wielded.
        
         | thriftwy wrote:
         | Since 50s we've got so many improvements in construction tech
         | that I would expect costs to plummet, instead they are higher
         | than ever. Countries use immigrant labor (sometimes illegal)
         | and have all tech available and the price only goes up. I
         | wonder why.
        
         | kaiju0 wrote:
         | Each build is unique and requires way too much overhead. The
         | new generation of preapproved factory assembled SMR's are the
         | future.
        
           | ortusdux wrote:
           | I have high hopes for prefab modular systems. The same design
           | approval covers 1000+ units, most designs fit on a flat-bed
           | for transport, and instillations can scale up as needed.
        
           | yvdriess wrote:
           | Having talked to a nuclear engineer about this: SMRs are
           | being politically pushed because of their political and
           | financing convenience, more than engineering reasons. Power
           | output scales really well with reactor size, so it makes much
           | more sense to build the one big expensive power plant than a
           | multitude of smaller ones. SMRs do make sense for off-grid or
           | on-site power, but not for grid electricity.
        
             | Ericson2314 wrote:
             | The ideal case is that SMRs are not the end goal, but a way
             | to rebuild the supply chain. As soon as we have SMRs in
             | prod, rather than building more of them, we should attempt
             | to increase the size of deployments with minimal falling
             | back on in-situ construction.
             | 
             | SMRs take the supply chain metaphor a bit too literally: we
             | do need practice but assembling prefabbed parts at a larger
             | scale is fine too. There is a spectrum of options and we
             | just need to avoid "special snowflake boondoggles".
             | 
             | Given the US's fucked NIMBY culture, it well may be that
             | SMRs are the best route despite these inefficiencies. Just
             | don't expect the "solar model" where we just shit out lots
             | of lousy product and that's it.
        
             | zbrozek wrote:
             | From a thermal physics and material science perspective,
             | yes, bigger is definitely better. But there's a lot to be
             | said of the value of being able to mass-produce a product
             | in a factory and ship it in nearly ready-to-use state to
             | its destination.
             | 
             | There would also be a lot of value in turning off the
             | ability of folks to NIMBY everything from new power plants
             | to housing.
        
             | petre wrote:
             | Alvin Weinberg begs to differ, at least concerning PWRs and
             | BWRs. The bigger plants are more efficient but less safe.
             | 
             | https://m.youtube.com/watch?v=iW8yuyk3Ugw
             | 
             | One can scale the size by using several reactors, which is
             | exactly what NuScale and others aim to do. One also doesn't
             | have an unavailability problem having to shut down a large
             | reactor in order to refuel it.
        
       | 01100011 wrote:
       | Totally unrelated question but does anyone know if nuclear
       | powered US Navy vessels are able to feed power to the grid while
       | in port? I know it's technically possible, just not sure if
       | anyone has ever implemented such a thing.
        
         | pydry wrote:
         | Russia did it https://thebarentsobserver.com/en/arctic-
         | industry-and-energy...
        
         | loeg wrote:
         | I think they do while in port in Hawaii. Hawaiian electricity
         | is largely diesel (other than nuclear coming from Navy ships).
        
           | jandrese wrote:
           | Isn't Hawaii also transitioning a significant fraction of its
           | generation to solar? Diesel can be ramped up and down easily
           | to account for varying production from solar.
        
             | db65edfc7996 wrote:
             | The Hawaii government page [0] list a goal of getting to
             | 100% renewables by 2045. If I am reading the report
             | correctly, the 2020 number was already at 36% renewables.
             | 
             | [0]: https://energy.hawaii.gov/wp-
             | content/uploads/2022/01/HSEO_20... ( pdf warning )
        
             | loeg wrote:
             | Yes, but I'm not sure what the current percentage is or how
             | fast that change is happening.
        
               | jandrese wrote:
               | Solar and Wind have really been exploding in the past few
               | years. I fully expect this hand wringing over nuclear
               | will end up being overtaken by events in the next 30
               | years or so. The real limitation at the moment is battery
               | technology, there is a lot riding on finding cheap and
               | efficient energy storage.
        
         | scoopertrooper wrote:
         | My understanding is that they shut down the reactors in port
         | and run off the grid.
        
           | ortusdux wrote:
           | This is my understanding as well. I believe that they may
           | even shut down at sea and run in on batteries/generators,
           | which doubles as a way to test those systems. Refueling is
           | quite the ordeal, so there is an incentive to minimize fuel
           | depletion.
        
             | dodobirdlord wrote:
             | Reactors may sometimes be shut down at sea to test backup
             | generators and run reactor restarting drills, but it would
             | not be done otherwise. Aircraft carrier propulsion comes
             | from steam from the reactors - if the reactors are offline
             | the ship cannot move. Moreover, the backup generators burn
             | jet fuel, which is convenient because aircraft carriers
             | already have a store of jet fuel for the planes and so
             | don't have to carry extra fuel for the backup generators,
             | but it's very expensive, and not something that would be
             | done outside of an emergency or testing emergency
             | preparedness.
        
             | nickelpro wrote:
             | Nuclear powered vessels do not spend any significant amount
             | of time shutdown at sea, and there is no reason to.
             | Electrical power is a very small fraction of their total
             | MWh production, with almost all power going to propulsion.
             | 
             | When fuel lifetime becomes an issue for a nuclear naval
             | vessel they will have propulsion limits in place that limit
             | transit speeds to those which are most efficient for the
             | propulsion turbines.
        
         | acidburnNSA wrote:
         | Sometimes, in relief missions like after big earthquakes.
         | 
         | More often they use the onboard desalination plants (also
         | nuclear powered) to make lots of fresh water.
         | 
         | e.g. https://www.militarynews.com/norfolk-navy-
         | flagship/oceana/ne...
        
         | colechristensen wrote:
         | The largest nuclear vessel produces up to 160 MW, the smallest
         | single nuclear reactor power plant about 500 MW (sites often
         | have multiples), the largest nuclear reactor in the US is about
         | 4 GW. An average wind turbine produces 2-3 MW.
        
           | 2OEH8eoCRo0 wrote:
           | CVN-78s reactor according to Wikipedia:
           | 
           | > It is estimated that the total thermal power output of the
           | A1B will be around 700 MW
           | 
           | CVN-78 has two of them.
        
             | comrh wrote:
             | I think the discrepancy is not all the thermal output is
             | converted to electricity:
             | 
             | > A1B reactors likely produce enough steam to generate 125
             | megawatts (168,000 hp) of electricity, plus 350,000 shaft
             | horsepower (260 MW) to power the four propeller shafts.
        
               | 2OEH8eoCRo0 wrote:
               | Ah, yes. Most steam is not converted to electricity.
               | Should have mentioned that.
        
         | jeffbee wrote:
         | A naval reactor produces most of its power in direct steam
         | propulsion and relatively little (~1/3rd) converted to
         | electricity. I think the largest vessels can generate just
         | 125MW which isn't much at all. The only beneficial use of a
         | navy vessel as mobile infrastructure that I can recall is when
         | the USS Carl Vinson was used to produce drinking water for
         | Haiti after their earthquake.
        
         | nickelpro wrote:
         | Navy nuclear vessels have (relative to the grid) little real
         | power generation capability and cannot handle grid reactive
         | loading at all. The grid appears as an infinite reactive load
         | to shipboard electrical switching equipment.
        
           | 01100011 wrote:
           | Sure, relative to the total grid capacity it's not much but
           | during peak loads every bit helps.
           | 
           | I suppose you'd want to transition from ship to shore via a
           | DC path and convert it back to synchronized AC which should
           | avoid issues with reactive loads.
        
         | hansel_der wrote:
         | quick google suggests it has been done in emergencies.
        
       | cvccvroomvroom wrote:
       | Insurance, licensure, and NIMBY pressure.
       | 
       | Previously in nuclear industry.
        
       | justinsb wrote:
       | The only nuclear plant under construction in the US is at Plant
       | Vogtle, in Georgia. Regulators set up a system (CWIP) whereby the
       | companies building the plant earn a 10% return on their costs,
       | until the plants come online. I think it's not surprising
       | therefore that costs keep increasing and the delays keep coming.
       | I don't think we can infer that nuclear power plants cannot be
       | built at reasonable cost, rather that we need to consider
       | "regulatory capture" as a significant construction risk.
       | 
       | (Some admittedly one-sided background on CWIP:
       | https://stopcwip.com/ )
        
       | ak217 wrote:
       | > Nuclear is sometimes praised for having lower fuel costs, but
       | all else being equal (ie: assuming total production cost stays
       | constant), it's better to have a larger fraction of your
       | electricity costs be variable, so that if demand drops then
       | production cost drops as well.
       | 
       | This is not obviously true. I could make an argument that base
       | load generation capacity (nuclear and hydro in particular) should
       | be state-owned or largely state-sponsored, both to avoid economic
       | price shocks/volatility and because it's good for national
       | security (check out what Europe is going through right now).
        
         | jacquesm wrote:
         | What Europe is going through right now is _mostly_ caused by
         | those large, state sponsored conglomerates. It 's the smaller
         | private operators that are doing just fine.
        
         | stingraycharles wrote:
         | Yeah this statement makes me think that OP had very little
         | actual knowledge about energy pricing.
         | 
         | If there's ever a complicated market of supply and demand it's
         | energy. Having a huge chunk of the supply be stable and
         | controllable is absolutely desired, will help simplify
         | operations a lot, make supply more predictable, and as a result
         | deliver more stable prices.
         | 
         | It's not as if the population is suddenly getting cheaper
         | windmills if there's too much supply; if the energy supplier is
         | losing money due to oversupply, they will need to get their
         | money back another way, so it's always the consumer that pays
         | anyway.
        
           | logifail wrote:
           | > Having a huge chunk of the supply be stable and
           | controllable
           | 
           | Nuclear might be stable, but "controllable"?
           | 
           | On the days when it's windy and sunny in your part of the
           | planet, try telling your local nuclear plants they're not
           | required.
           | 
           | There's simply no reason to guarantee nuclear generators a
           | fixed electricity price decades in advance, like the UK did
           | with Hinkley Point.[0]
           | 
           | [0] https://www.theguardian.com/news/2017/dec/21/hinkley-
           | point-c...
        
             | dTal wrote:
             | I don't know why you say you can't throttle nuclear up and
             | down. All thermal plants have inertia, but nuclear is if
             | anything easier to throttle than fossil fuels.
             | 
             | There's a lot of numbers, and comments from nuclear plant
             | operators, in this Reddit thread: https://www.reddit.com/r/
             | NuclearPower/comments/m0rwso/how_fa...
        
               | logifail wrote:
               | > I don't know why you say you can't throttle nuclear up
               | and down. All thermal plants have inertia, but nuclear is
               | if anything easier to throttle than fossil fuels.
               | 
               | In that case why don't we let the market build nuclear
               | plants without any state guarantees or insurance and they
               | can simply "throttle up" when they're required. No need
               | to fix a strike price for decades before investors are
               | interested.
               | 
               | I'm sure it makes financial sense. Honest. /s
        
               | dTal wrote:
               | That's quite the non-sequitor. An exploration of how
               | broken "the market" is would be an entirely separate
               | discussion. If you wanted to have that discussion, we
               | could start by analyzing the massive subsidies enjoyed by
               | fossil fuels, not to mention the complete lack of
               | accounting for its toxic byproducts which are dumped into
               | the atmosphere (while nuclear power is legally required
               | to track and safely store every molecule).
               | 
               | Or we could just keep building gas peaker plants and
               | ignore the mass die-offs, because "the market" can't
               | possibly be wrong, right? /s
        
               | akvadrako wrote:
               | Nuclear has very low fuel costs, under 1%. So it really
               | doesnt make sense to throttle them unless the price goes
               | negative.
               | 
               | Having an upfront guaranteed price is just a way to
               | spread the construction cost over more time.
        
           | danielmarkbruce wrote:
           | "all else equal" is in there.
           | 
           | He was trying to make a point - that flexibility is valuable.
           | And it is valuable. _All else equal_ you 'd take more
           | flexibility than less, especially since demand moves around a
           | good amount.
        
             | stingraycharles wrote:
             | But it isn't flexibility we can control. It's either
             | fluctuations in supply which we need to absorb somehow, or
             | it's scaling down supply. The big differentiator is the
             | flexibility to scale _up_ supply when you need it, and it's
             | precisely this flexibility that's missing.
        
         | logifail wrote:
         | > base load generation capacity
         | 
         | What's the consensus on the meaning of 'base load generation
         | capacity'?
         | 
         | There are those who'd say it's an archaic term often used to
         | defend power sources that can't ramp up and down to meet
         | demand, and nuclear would be top of that list. If - for
         | instance - there are times when it's windy _and_ sunny, why
         | should consumers have to pay more than the market rate to
         | nuclear generators, just because nuclear is inflexible?
         | 
         | More broadly: what's the actual use case for 'base load
         | generation capacity' over the coming decades?
        
           | mschuster91 wrote:
           | > More broadly: what's the actual use case for 'base load
           | generation capacity' over the coming decades?
           | 
           | The use case is that there should be enough capacity under
           | governmental control to ensure that even in a case of crisis
           | (such as, say, an oil price hike, a war or import blockades)
           | the base load of the citizenry is still accounted for - big
           | industries might be temporarily restricted, but no citizen
           | should freeze in winter because the forces of the market deem
           | it more profitable to have some large company buy their way
           | out.
        
             | logifail wrote:
             | > there should be enough capacity under governmental
             | control [..]
             | 
             | Q: You really want to nationalize power generation?
        
           | Paradigma11 wrote:
           | Use Case: You pay more to get a reliable service for the same
           | price regardless of weather conditions.
        
           | [deleted]
        
           | orangeoxidation wrote:
           | Yup, that seems a strange argument. Power usage varies during
           | the day, but it doesn't really go below some "base load".
           | 
           | Nuclear can run basically 24/7, but you cannot turn it on or
           | off quick enough to to react to hourly changes in demand. So
           | nuclear power is only good for base load.
           | 
           | We need peaker plants to get the rest. Gas plants are the
           | popular (cheap) choice for this. Carbon free alternatives are
           | pumped hydro or batteries.
           | 
           | With renewables the production capacity is variable as well.
           | 
           | To fill possible "holes" in it we don't need more base load.
           | What renewables need are ... peaker plants. A role nuclear
           | reactors are exceptionally unsuited to fill.
        
             | logifail wrote:
             | > you cannot turn it on or off quick enough to to react to
             | hourly changes in demand
             | 
             | Surely you'd want actually want suppliers to react to
             | changes in spot price, not just demand? If it's windy and
             | sunny, it might not matter if demand is high! If it's calm
             | and cloudy, you have a problem.
             | 
             | If the spot electricity price is high, you want providers
             | to jump in and supply electricity. If it's low, or indeed
             | goes negative[0], you want them to shut down.
             | 
             | Nuclear just doesn't fit this model, since investors appear
             | to want the strike price guaranteed for several decades
             | before they'll even start pouring concrete for their plant.
             | 
             | [0] https://www.sciencedirect.com/science/article/pii/S2666
             | 79242...
        
             | masklinn wrote:
             | > Nuclear can run basically 24/7, but you cannot turn it on
             | or off quick enough to to react to hourly changes in
             | demand. So nuclear power is only good for base load.
             | 
             | That's not exactly true, you can build nuclear plants for
             | load following which provides some amount of flexibility,
             | at the cost of some efficiency (about 1% I think).
             | 
             | IIRC French plants can operate between 30 and 100% rated
             | power, and ramp rates can reach 5% per minute (though
             | normal rates are 1 to 3). French nukes regularly have to
             | ramp up and down quickly to compensate for wind variation
             | and monday pickup (electricity consumptions goes way down
             | over the weekend, especially nice spring weekends, then
             | back way up on week start).
        
             | SiempreViernes wrote:
             | > but you cannot turn it on or off quick enough to to react
             | to hourly changes in demand
             | 
             | To be precise, it cannot _stop consuming fuel_ quick enough
             | that there 's any important savings, and so nuclear power
             | plants always want to run at full capacity because
             | otherwise they are wasting fuel.
             | 
             | But more than that, fuel consumption is simply not a very
             | big part of the running cost of a nuclear power plant, most
             | is fixed cost so that even if they could change the fuel
             | consumption quickly there's just not enough savings to
             | really bother with it.
        
               | logifail wrote:
               | > To be precise, it cannot stop consuming fuel quick
               | enough that there's any important savings, and so nuclear
               | power plants always want to run at full capacity because
               | otherwise they are wasting fuel. But more than that, fuel
               | consumption is simply not a very big part of the running
               | cost of a nuclear power plant, most is fixed cost so that
               | even if they could change the fuel consumption quickly
               | there's just not enough savings to really bother with it
               | 
               | (Sorry) but that sounds like a slightly long-winded way
               | of saying that nuclear just isn't economically viable.
               | 
               | "the costs of renewables continue to fall due to
               | incremental manufacturing and installation improvements
               | while nuclear, despite over half a century of industrial
               | experience, continues to see costs rising"[0]
               | 
               | [0] https://www.pv-magazine.com/2020/09/24/nuclear-power-
               | is-now-...
        
           | necheffa wrote:
           | The technology to do load follow at a nuclear plant exists.
           | 
           | If you think about it, "base load" has nothing to do with the
           | power source. Given some period of time, say a day, you are
           | always going to have a certain minimum demand for power
           | generation in a geographic location. Congratulations, you
           | have identified base load.
           | 
           | As the grid becomes more distributed and therefore less
           | centralized, you are going to see base load hitting lower
           | peaks because individual power generation stations will have
           | less aggregate demand. But until society reaches a point
           | where at least part of the day there is zero demand on the
           | grid (fat chance) you will always have base load in some
           | shape or form.
        
             | logifail wrote:
             | > you are always going to have a certain minimum demand for
             | power generation in a geographic location. Congratulations,
             | you have identified base load.
             | 
             | If it's - say - sunny and windy, your renewables are always
             | going to undercut _all_ other generators. So when they
             | undercut nuclear, basic market forces should mean nuclear
             | doesn 't get to supply a single MW, and if that means
             | investors lose out, well, tough.
             | 
             | Base load should _always_ be supplied by the cheapest
             | supplier. Not the least flexible and /or the ones with the
             | highest fixed costs.
        
           | fallingknife wrote:
           | I tend to disbelieve the people who say it's unnecessary
           | because there are many places where the cheapest power
           | generation has come from renewables for quite some time, and
           | yet all of those places still have base load on the grid.
        
             | jacquesm wrote:
             | Baseload has several major ingredients: the required
             | continuous consumption, connectivity of areas that are
             | remote from each other where the one has a surplus and the
             | other a deficit, local overcapacity storage options and
             | installed capacity from 'guaranteed' sources (and no source
             | is 100% guaranteed, typically even the most stable sources
             | are down 20 to 40% of the time for maintenance, refueling,
             | repairs and so on).
             | 
             | The required baseload is then further influenced by load
             | variability, and rate-of-change. Not all generation
             | equipment can spin up / down equally fast, and sometimes
             | the effect of for instance a shut-down is that it will take
             | a long time to go back online.
             | 
             | Baseload is a function of a whole interconnected grid
             | rather than of some locality, and this is a big difference
             | between how laypeople see this and how people in the power
             | business see it. It's not as if the electrons that are
             | pushed into a wire in say Southern France need to get all
             | the way to Poland to light a bulb there, all that the
             | various generators do is maintain their local grid by
             | making available enough power locally that lightbulbs in
             | France are served by their local power stations and
             | lightbulbs in Poland are served by theirs. This minimizes
             | transmission losses.
             | 
             | If you have a surplus and the distance is large then with
             | conventional (AC) transmission lines there is an upper
             | limit to how big an area you can serve before the losses
             | make that no longer economical. HVDC has nicer properties
             | for long distance transmission which has some very
             | interesting consequences for baseload: suddenly wind and
             | solar thousands of KM (multiple timezones) away can be used
             | to provide power to some locality, reducing the need for
             | local generation capacity if the price is right.
             | 
             | This revolution is happening right now, the HVDC grid
             | interconnects are shaping up rapidly with more and more of
             | these coming on-line. Especially the longer East-West runs
             | have the possibility to materially affect the amount of
             | fossil/nuclear required for when solar and wind are
             | insufficient, as well as the North-South ones from areas
             | where there is a lot of hydro generation capacity.
        
         | danielmarkbruce wrote:
         | The statement is obviously true.
         | 
         | The part which makes it so is "all else being equal". He is
         | just saying "flexibility is more valuable than it might seem on
         | the surface, because demand moves around".
        
       | mbostleman wrote:
       | Doubling the cost consequence of regulations, regulations
       | changing causing in-progress projects to go back and remove and
       | re-do work, negative learning - all of these things are a result
       | of our society not gathering around the mission. If we wanted it,
       | we would fix all these things. But it just doesn't have support.
       | It seems like an incredible tragedy that is at least proportional
       | to that of climate change since from most reasonable projections,
       | nuclear - assuming the technological challenges can be solved -
       | is the quickest way to reduce our impact on climate change. The
       | fact that we insist on suppressing the courage to solve the
       | problems makes me question the integrity of those who run the
       | narratives on climate change policy.
        
       | lelag wrote:
       | The article misses another important aspect: loss of nuclear
       | competency.
       | 
       | Between the 60s and 80s, there were many nuclear reactors
       | projects which allowed an industry to develop and get better at
       | it.
       | 
       | But since the 80s, there was comparatively very few new reactors
       | built for over 20-30 years. The workforce that had the skills and
       | knowledge related to actually building nuclear plants had mostly
       | retired and their replacement had only theoretical knowledge and
       | no actual experience. This makes building new reactor much harder
       | than it should be.
       | 
       | The embattled EPR project at Flammanville is an exemple of that:
       | the specialised company that was hired to forge the nuclear
       | vessel were simply unable to build to spec and a defective
       | critical piece was delivered, creating delays and cost increases.
       | In the end, they even had to use it anyway as it was not
       | economically feasible to simply have another one made.
        
         | colinmhayes wrote:
         | This is undoubtedly true, but south korea has now been building
         | nuclear plants for decades, and their costs haven't really
         | dropped at all.
        
           | ImHereToVote wrote:
           | According to the article they have
        
             | alex_young wrote:
             | From the article:                 > The only country where
             | the costs of nuclear plant construction seem to have
             | steadily decreased is South Korea:            > The fact
             | that South Korea is the only country to exhibit this trend
             | has led some experts to speculate that the cost data (which
             | comes directly from the utility and hasn't been
             | independently audited) has been manipulated and we
             | shouldn't draw conclusions from it.
        
               | bhc wrote:
               | The government that was in power from 2017 to 2022 put a
               | moratorium on new reactor construction there and promised
               | a full phase-out, and although the newly-elected
               | government promised to reverse this, it likely has done
               | some damage to S. Korea's civilian nuclear capabilities.
        
               | DennisP wrote:
               | In absolute terms, Japan and India have costs similar to
               | South Korea's.
               | 
               | https://www.sciencedirect.com/science/article/pii/S030142
               | 151...
               | 
               | See figure 12 for a quick overview. And from the
               | introduction: "In contrast to the rapid cost escalation
               | that characterized nuclear construction in the United
               | States, we find evidence of much milder cost escalation
               | in many countries, including absolute cost declines in
               | some countries and specific eras. Our new findings
               | suggest that there is no inherent cost escalation trend
               | associated with nuclear technology."
        
               | danans wrote:
               | > In absolute terms, Japan and India have costs similar
               | to South Korea
               | 
               | That countries like Japan and Korea have similar absolute
               | costs as India which has a 4-5x lower PPP adjusted GDP
               | per capita suggests that the price for labor for nuclear
               | power plant construction is globally set (relatively few
               | qualified engineers who can demand a high price), or
               | India uses a lot more labor, or a combination of both.
        
               | 8ytecoder wrote:
               | India probably uses a lot of foreign expertise. It's
               | either the French or the Russians that supply the
               | reactor. That would change in the future I suppose and
               | cost would drop. Also, corruption.
        
               | mandevil wrote:
               | Japan ceased all construction of nuclear reactors in
               | response to Fukushima daiichi, and since that was a
               | decade ago I'm betting that all the competence they built
               | up has disappeared.
               | 
               | In other words, very similar to what happened in the US
               | in response to Three Mile Island: after a scary nuclear
               | incident there was a lengthy pause in nuclear
               | construction which meant that all of the skills and
               | learning-by-doing that had accumulated up to that point
               | went away, and starting again would be significantly more
               | expensive and subject to massive schedule and cost
               | overruns.
        
               | jandrese wrote:
               | If anything this seems to support the position that the
               | only way to reduce costs is to increase volume. A classic
               | economies of scale example. Instead the experts want to
               | disregard the data for vague reasons.
        
               | Retric wrote:
               | South Korea, Japan, and India all have similar costs
               | which suggests South Korea isn't benefiting significantly
               | from continuous construction.
               | 
               | Economies of scale generally exist, but it's not magic. A
               | large fraction of construction costs for nuclear power
               | plants is very similar to other structures. A high
               | pressure steam pipe is a high pressure steam pipe and
               | people are constantly building structures using them.
        
               | clairity wrote:
               | also, quantity is the (primary) independent variable in
               | economies of scale, and at quantities of dozens for
               | nuclear plants, you can't get much economies, as opposed
               | to when quantities are in the many thousands/millions.
        
               | trashtester wrote:
               | The cost of flying has come down by 50% since 1980, and
               | while an airplane is a simpler machine than a nuclear
               | plant, the two industries also have a lot in common (such
               | as the perception of risk not being in their favour).
               | 
               | By doing international standardization and coordination
               | in ways similar to the aircraft industry, the same should
               | be possible for the nuclear power industry.
               | 
               | It should be possible to consolidate most of global
               | production down to a handful of companies (like Boing and
               | Airbus), with a forest of subcontractors in the same way
               | that was done for airliners, and achieve similar
               | economies of scale.
               | 
               | Successful designs could be re-used over a period of 20
               | years or more, with only minor modernizations of things
               | like electronics, like the Airbus A320 or Boing 747.
               | 
               | Ideally, we should have done this in 1980, but even if we
               | start today, nuclear can provide a lot of energy at very
               | competitive prices in the next 60-100 years. By then, we
               | should have fusion or the ability to build energy storage
               | cheeply enough to make renewables (probably solar)
               | competitive.
        
               | sanxiyn wrote:
               | By the way, this blog (Construction Physics) is about why
               | construction in general (not nuclear power construction
               | in particular) is expensive. One big part is that
               | construction is done on site, and site-to-site variation
               | hurts standardization and economies of scale.
               | 
               | Finished airplanes can transport itself by flying. This
               | advantage is particular to aircraft industry and probably
               | can't be copied by other industries. Finished buildings
               | can't transport itself.
        
               | treme wrote:
               | Koreans excel in cost-efficient construction projects.
               | Korean companies are often considered for best bang-for-
               | buck value when developing countries are interested in
               | big infrastructure projects nowadays.
               | 
               | https://en.wikipedia.org/wiki/1915_%C3%87anakkale_Bridge
               | 
               | https://en.wikipedia.org/wiki/Burj_Khalifa
               | 
               | https://en.wikipedia.org/wiki/Bataan%E2%80%93Cavite_Inter
               | lin...
        
             | [deleted]
        
           | pfdietz wrote:
           | They've built four NPPs in the UAE that may end up producing
           | at $0.08/kWh.
           | 
           | Unfortunately, UAE is also building PV that will be producing
           | at $0.013/kWh. And since they're still burning gas for most
           | of their power, every kWh from solar goes straight to
           | reducing the overall cost and CO2 emission, five times
           | cheaper than the NPPs will.
        
         | samstave wrote:
         | My grandfather was a Nuclear Engineer for General Electric his
         | whole life (worked like 60 years at GE - he was one of the
         | designers of Hanford.
         | 
         | He died of cancer, thyroid cancer of exenguination (bleeding
         | out of your mouth)
         | 
         | My grandmother received a fairly large settlement from the
         | class action lawsuit against GE for exposing engineers to
         | radiation for decades without proper safety...
        
           | ratsmack wrote:
           | My uncle worked at Hanford his entire life but his last few
           | years of life were not good. He he retired with numerous
           | health issues and was essentially a mental vegetable the last
           | few years of his life. He was at Hanford from it's inception
           | up to around the seventies when he retired.
        
             | cco wrote:
             | Son of a Hanford man here. I believe my father started work
             | at Hanford _after_ the bulk of their issues were wrapped up
             | but I'm sure there was some increased exposure relative to
             | background.
             | 
             | Sorry to hear about your uncle's experience, a lot of pain
             | came out of that facility.
        
             | samstave wrote:
             | My grandfather was Kenneth Victor Stave. If that name means
             | anything to your family.
             | 
             | Also, I dont know if your uncle was part of that suit
             | against GE - but it may be something you want to look up.
             | 
             | I don't have any further info to provide on the subject, my
             | grandfather passed in 1996. My grandmother last year. So I
             | cant ask anyone...
        
         | AtlasBarfed wrote:
         | I think this is actually a good thing.
         | 
         | The old designs are dangerous, expensive, and wasteful. The
         | regulatory, economic, and political environment that resulted
         | in their design ultimately resulted in reactors run without
         | proper controls, supervision, or long term safety. The
         | resulting waste was not properly considered from a life cycle
         | perspective.
         | 
         | Disclaimer: I'm not a nuclear expert, but man I loved those
         | LFTR presentations. What really appeals to me about LFTR is the
         | inherent safety, the near-full use of fuel, and the
         | scalability. I understand there are challenges for the
         | materials and containment, but I believe the smaller size of
         | the reactor can lend itself to replacement and manufacturing.
         | 
         | So a "clean slate" with new people, regulations, standards,
         | expectations, computer simulation, and lifecycle planning would
         | do nuclear a huge bonus.
         | 
         | But ultimately it doesn't matter. It won't be price competitive
         | with wind/solar and can't even target a 10-year price point
         | with the wind/solar improvement curves. Same issue the fusion
         | story on the front page faces.
         | 
         | Let's continue active research, but commercialization is a
         | waste of time and money right now. When wind/solar stabilize
         | their cost curves, then nuclear (or fusion) will have something
         | to target commercially. IF they can get there.
        
         | nomel wrote:
         | > The article misses another important aspect
         | 
         | I don't think that's a fair claim, considering "Part I" is in
         | the title.
        
         | sky-kedge0749 wrote:
         | I'm just riffing here but this doesn't seem like an
         | insurmountable problem if you're willing to spend. Open up a
         | training school, put the old guard in as instructors, get some
         | good students, pay everyone big money. Build a lab reactor for
         | hands-on practice, and pay to put students as glorified interns
         | into under-construction and operating plants across the world.
         | A few years later, you've got your people.
         | 
         | I don't mean to say it would be trivial but it seems like you
         | could do the whole thing for a couple billion USD a year.
        
         | masklinn wrote:
         | TBF Flamanville 3 was a shitshow from top to bottom, starting
         | from anyone actually taking Areva's completely unrealistic
         | timeframes seriously: the claim was something like 3 years for
         | the build, EDF assumed production within 4.5 years.
         | 
         | For a novel build of a barely finished design.
         | 
         | 4.5 years is probably the shortest time it took to build a CP
         | (900MW) reactor at the height of France's reactor-building
         | frenzy (St-Laurent-B-1 construction started in May 1976 and
         | ended in January 1981, 4 years and 8 months).
         | 
         | The next generation (P4) I don't think any took less than 6
         | years to build, and the embattled N4 generation immediately
         | preceding the EPR the first reactor (Chooz 1, of only 4) had a
         | build time of _12 years_ (and 7 months), the last (and
         | fastest), Civaux 2, being completed in a  "mere" 8 ( and 8
         | months).
         | 
         | And the N4 had and still has significant teething issues: soon
         | after they were put into production they suffered from leaks in
         | cooling pipes leading to all 4 being stopped for 10 months, and
         | at the 10 years revision in 2021 extensive stress corrosion
         | cracking of the primary circuit was discovered, all the N4s
         | have been stopped and the last news are they won't be restarted
         | until 2023.
        
         | epistasis wrote:
         | The recent US nuclear construction projects have been plagued
         | with similar incompetence, such as trying to build plans that
         | were unconstructable, and then having to get regulatory
         | approval for the changes.
         | 
         | I would like to see comparisons to other large construction
         | projects too. The US is really really bad at large construction
         | projects, but European construction seems a lot better.
        
           | mandevil wrote:
           | Unfortunately, when it comes to nuclear Europe's (excluding
           | Russia) basically in the same boat as the US.
           | 
           | Consulting the current list of nuclear power plants under
           | construction around the world by a pro-nuclear power group
           | here: https://world-nuclear.org/information-library/current-
           | and-fu...
           | 
           | I see the following as the only ones still under construction
           | in all of Europe (not counting Belarus and Russia): 1)
           | Mochovce 3 in Slovakia. Construction started November 2008,
           | originally scheduled to complete in 2012, now hopefully
           | complete later this year, so 15 years total, 10 years late.
           | 2) Flamenville 3 in France. Construction started in 2007,
           | originally scheduled to complete in 2012. Hopefully complete
           | in 2023, so 16 years later, 11 years late. 3) Mochovce 4 in
           | Slovakia. Construction started November 2008, original
           | scheduled to complete in 2013, now hopefully complete in
           | 2023, so 16 years total, 10 years late. 4,5) Hinkley Point C1
           | and C2 in the UK. Construction started in roughly 2008,
           | originally expected to be online 2022 or so ("early 2020s" is
           | the best I can find with Google now, and that's for both C1
           | and C2 to be online). Now C1 is expected to be complete in
           | 2027, and C2 in 2028. So 19-20 years total, 6 years late.
           | 
           | (The US has two reactors on the list, Vogtle 3 and 4, started
           | in 2009, originally expected to finish in 2016 and 2017, now
           | expected to finish in 2023.)
           | 
           | I suspect that Europe's success in building rapid transit,
           | compared to America, is due to the fact that they were
           | continuously building such systems, whereas the US largely
           | hasn't, so there is no cohort of engineers and workers who
           | learned-by-doing and get better over time. But in nuclear,
           | those workers seem to have gone in Europe as well- you can
           | see from this 2020 chart (https://en.wikipedia.org/wiki/Nucle
           | ar_power_in_France#/media...) that France built almost all of
           | their reactors in a giant lump between 1970 and 1983, built a
           | few reactors later in the 1980s (presumably late career work
           | from the people who had built so many earlier), and has found
           | building a new reactor to be really hard, e.g. Flamenville 3
           | is just as big a disaster as Vogtle.
        
             | thow-58d4e8b wrote:
             | To add one more data point - Olkiluoto 3, started 2005, was
             | expected to finish in 2009. Completed late 2021, 12 years
             | late
             | 
             | Cherry on top - after producing electricity for about a
             | week, it had to shut down for another 3 months. Then, after
             | ramping up to about 30% of the capacity, it encountered
             | another problem, delaying it for another 5 months. Here we
             | are in June 2022, 17 years later, Olkiluoto 3 provides
             | exactly 0 MW to the Finnish grid
        
             | spc476 wrote:
             | I also recall reading (somewhere) that France basically
             | ended up with two nuclear power plan designs used
             | repeatedly, unlike in the US where nearly every nuclear
             | power plant is unique. That might account for the lower
             | costs shown in the article.
        
               | mandevil wrote:
               | That is true for the 1970's and 1980's boom of production
               | in France, but is not true at present: the EPR they are
               | building at Flamenville 3 and Hinkley Point C1+C2 are the
               | sum total of those reactors currently under construction,
               | and none are currently operational, so those three are
               | likely to be the total number ever built.
               | 
               | It is true that the 34 CPY reactors, and the 20 P4
               | reactors, were produced in large enough numbers to create
               | a skilled class of workers and engineers who were deeply
               | experienced with building these reactors, but right now
               | all of those workers are retired.
               | 
               | And honestly, from observation, it appears that
               | rebuilding competence like this is a lot harder than
               | building it in the first place: when you are building the
               | first time everyone- the general public, the regulators,
               | the workers themselves- are more forgiving. When you've
               | lost that capacity and are trying to rebuild it you have
               | expectations set for a mature industry, but the skills
               | aren't there to deliver it.
        
           | krylon wrote:
           | > European construction seems a lot better
           | 
           | Unless the situation in the US is really, really _REALLY_
           | horrible, I doubt that. I don 't remember where, but one
           | country is in the process of building a nuclear power plant,
           | that is AFAIK unfinished, but already took way more time and
           | money than originally planned.
           | 
           | Doesn't even have to be nuclear - ask the Internet about
           | Hamburg's Elbphilharmonie or Stuttgart's train station.
           | Having public construction projects overrun their schedules
           | and budgets is a well-honored tradition, at least in Germany,
           | but I suspect our neighbors have similar customs.
        
             | mandevil wrote:
             | An excellent (English language) podcast about Berlin's
             | fiasco of an airport, BER:
             | https://www.radiospaetkauf.com/ber/ (29 years from planning
             | to completion, 14 years from construction start to opening,
             | 9 years late, budget from 1 billion Euro to almost 6).
             | 
             | One of the points they made in the podcast was similar to
             | TFA's: changes in construction are really expensive and
             | blow things out in costs. A new mayor came in and demanded
             | major changes once construction was underway in Berlin. And
             | then, when people said "this will cause problems" his
             | response was basically "we're Germany, we are the best at
             | planning, building and constructing, of course we can
             | handle this with no problems"...
        
             | ajmurmann wrote:
             | US is absolutely terrible at large infrastructure projects.
             | This article has some good details and statistics:
             | https://www.niskanencenter.org/wp-
             | content/uploads/2021/03/le...
             | 
             | In general, Alon Levy's blog has great articles on this
             | topic and he lives in Berlin
             | https://pedestrianobservations.com/
        
           | stefanfisk wrote:
           | I'd love to read more about this! do you have any tip on
           | where to start?
        
             | epistasis wrote:
             | Or if you were taking specifically about the nuclear
             | construction incompetence, the loca newspapers in South
             | Carolina and Georgia have been providing the best
             | reporting. Here's South Carolina's archive:
             | 
             | https://www.postandcourier.com/business/vc_summer_nuclear_p
             | r...
             | 
             | Search for "Georgia recorder vogtle" to get some of the
             | reports from the Georgia construction delays.
        
             | epistasis wrote:
             | The Pedestrian Observations blog is great for construction
             | cost analysis:
             | 
             | https://pedestrianobservations.com/
        
         | intrasight wrote:
         | My first job was doing software in the nuclear industry. Was
         | the late 80s. Probably the best job I ever had in terms of
         | working with extremely competent engineers. But they were all
         | in their 50s and 60s. After TMI, a generation of engineers said
         | "no" to nuclear careers. We can only imagine the alternate
         | history where that accident hadn't occurred.
        
         | jacquesm wrote:
         | I find it disconcerting that a defective critical piece ended
         | up being used anyway, regardless of the economies involved.
         | That might mean you have no reactor, but you can't just go an
         | substitute broken or out of spec parts for good ones.
        
           | otter-rock wrote:
           | Out-of-spec doesn't mean it can't work. It just means you
           | have to redo the design using what's effectively a different
           | part than you originally planned on.
        
             | jacquesm wrote:
             | Yes, but the idea here was to construct a nuclear power
             | plant, not to build a large pot for boiling soup in. You
             | can't take a critical component like that, spec it and then
             | suddenly pretend the spec never mattered in the first
             | place, then you have to admit that you're just winging it.
             | Changing the spec of the reactor vessel essentially
             | translates into a complete redesign of the reactor itself
             | unless you are willing to compromise on other aspects, such
             | as safety, longevity and so on.
             | 
             | We're not talking about a bracket here. Or an O-ring. When
             | was the last time something as stupid as an O-ring decided
             | the fate of... oh, never mind.
        
               | otter-rock wrote:
               | It's a requirements change. Those happen all the time in
               | everything. Why would that be completely forbidden or
               | impossible here? Like you said, it might really suck. But
               | the NRC does not allow trading off safety the way you
               | suggest.
        
               | jacquesm wrote:
               | Requirements changes are not driven by one-off material
               | defects in critical pieces of hardware.
               | 
               | That's simply a bending of the rules for economic
               | reasons. And it is one of the main reasons for me to
               | oppose nuclear: the fact that people will be people and
               | that at the root of every one of those disaster and near
               | disasters there was someone who thought they could get
               | away with something. We are ill equipped to deal with
               | this kind of responsibility, especially across a
               | timeframe measures in decades.
               | 
               | At the same time I would love to see us solve the climate
               | change problem, and I recognize that we will likely have
               | a nuclear component in there. But it will have to be done
               | by the book or we'll end up regretting it - again.
               | 
               | If we're going to start out with the normalization of
               | deviance on the #1 critical component of a reactor then I
               | think we are on the wrong path:
               | 
               | https://becht.com/becht-blog/entry/normalization-of-
               | deviance...
        
           | curiousllama wrote:
           | I mean depends how it's broken, right? Broken could just mean
           | anything from "will blow up momentarily" to "more inefficient
           | than spec, but totally safe"
        
             | sveme wrote:
             | But with that position any spec is useless.
        
             | immmmmm wrote:
             | Carbon migration problems during forging if i recall
             | correctly. So steel is out of specs. How badly will
             | certainly remain secret, like most things in this industry.
        
         | rob_c wrote:
         | yes, well said.
         | 
         | It's a shame the idiotic "green" movement after chernobyl is
         | rather annoying that it has set energy production in developed
         | nations back ~50 years and caused so much climate damage in the
         | mean time... but hey 'radioactive waste is corporate greed
         | maaannnn'.....
        
           | AtlasBarfed wrote:
           | But the green movement was ultimately correct. They didn't
           | know why and had nonsense arguments, but the fact is that
           | "old nuclear" was developed with insufficient long-term
           | safety. Fukushima showed that.
           | 
           | What also seems true is that you can't trust a company to run
           | them properly, no matter the regulations and audits. TEPCO
           | showed that. From people I know who've dealt with the nuclear
           | industry, there is a strong contempt of regulation in
           | sentiment/culture, likely due to the annoyances and perceived
           | costs.
           | 
           | This contempt however breeds a long term apathy towards
           | safety and maintenance. It's human nature.
           | 
           | There are reactor designs that are inherently meltdown proof
           | (LFTR) and use almost all their nuclear fuel (LFTR) and can,
           | I believe, breed old nuclear waste into usable fuel (LFTR).
           | They scale down to small closet sizes (LFTR) and so can be
           | more economically flexible. I believe pebble bed and others
           | can do similar things. LFTR allegedly can be designed to be
           | proliferation resistant, although I've seen opposing views
           | from much better educated people.
           | 
           | But the LFTR goals should be the standard of the nuclear
           | industry for next-gen. Not these massive solid rod huge dome
           | boondoggle-prone eyesores.
        
         | bsedlm wrote:
         | I think there's a larger point around the general notion of
         | "spread of competency".
         | 
         | the competency is not allowed to spread. there's a thick shroud
         | of secrecy around how all this sophisticated technology comes
         | about.
         | 
         | This is also why semiconductors are so difficult.
         | 
         | Back in the early 20th century the nuclear stuff was secret so
         | the nazis and then the russians would not get it.
         | 
         | Now semiconductors are also closely related to national
         | security stuff (china and taiwan). I find it quite suggestive
         | that most of the tech used to make the semiconductors is owned
         | european companies.
         | 
         | Finally, I have a sensation that in the 18-19th century it was
         | the chemical sciences that were similarly shrouded in secrecy
         | of this sort.
         | 
         | There was a topic here on HN the other day about how there's so
         | little popularization of chemistry... IMO, this is why, the
         | legacy of secrecy so to guarantee competitive industrial
         | advantages still casts its shadow.
        
           | djtango wrote:
           | At least in the UK, I found that salaries for Chemists were
           | depressingly low - PhD grads would earn around 30k GBP which
           | made it hard for me to justify studying for so long.
           | 
           | I really liked Chemistry but ended up moving into software
           | instead.
        
           | bsder wrote:
           | > the competency is not allowed to spread. there's a thick
           | shroud of secrecy around how all this sophisticated
           | technology comes about. > > This is also why semiconductors
           | are so difficult.
           | 
           | The dirty secret is that _all_ factories are hard to build
           | because nobody knows all the details to make them work.
           | 
           | It's that simple.
           | 
           | People bring operative knowledge to bear in the running of a
           | factory. Over time, that knowledge becomes baked into the
           | procedures, equipment, maintenance and people.
           | 
           | This is, in my opinion, something that everybody overlooks
           | about nuclear. Power plants and factories _need_ to evolve
           | and optimize over time to be successful.
           | 
           | Nuclear plants get encased in amber and can't do that. I
           | understand why people don't want to allow that. However, I
           | really think that this inability to evolve will doom _any_
           | large scale nuclear reactor design. Probably the only way
           | that nuclear becomes successful is very small, semi-sealed
           | power plants as the whole plant evolves at the manufacturing
           | facility rather than at the site.
           | 
           | > I find it quite suggestive that most of the tech used to
           | make the semiconductors is owned european companies.
           | 
           | This is hardly surprising. They're spinouts of the big
           | conglomerates from the 1980s (ASML is from Philips, no?).
           | These conglomerates _didn 't exist_ in Japan (maybe--MITI was
           | funding the hell out of things in Japan in the early 1980s so
           | my memory may be off), China, etc. back when this stuff was
           | getting started and spun out.
           | 
           |  _TSMC_ is actually the anomaly. It took a _very_ determined
           | effort with a lot of money being shoveled around by the
           | government combined with a disgruntled TI executive of
           | Chinese background and all of his knowledge and contacts to
           | put it all together.
        
           | bumby wrote:
           | > _I have a sensation that in the 18-19th century it was the
           | chemical sciences that were similarly shrouded in secrecy._
           | 
           | I'm not sure this is the case. Chemistry and geology were
           | both popular with hobbyists (albeit, it seemed to often be
           | aristocratic hobbyists) during that period.
        
           | selimthegrim wrote:
           | Link to the topic?
        
             | bsedlm wrote:
             | https://news.ycombinator.com/item?id=31648981
        
         | brandmeyer wrote:
         | This is one of the reasons for continuing to incrementally
         | design and build new submarine and aircraft carrier reactors.
         | If the expertise is to re-emerge in the US commercial sector,
         | it may require another cross-pollination effort from the
         | military.
         | 
         | The difficulty is that both military reactors and commercial
         | power reactors have evolved considerably since their initial
         | branch point. Commercial power reactors provide base load (run
         | at full power) for a year or two and then get refueled.
         | Military reactors now last the life of the ship without
         | refueling at all, but are optimized for propulsion's variable
         | demands.
        
           | nixonpjoshua1 wrote:
           | Perhaps military style reactors designed for propulsion loads
           | would be a good match for balancing renewables on the grid as
           | an alternative to natural gas peaker plants
        
         | rndmind wrote:
         | Consider how new the industry is, this comment is an hilarious
         | load of logical fallacies
        
       | Oarch wrote:
       | I'm enjoy HN's ongoing obsession with this substack.
       | 
       | Seeing it regularly appear here lets me forget for a moment that
       | my industry (construction) is still in the technological Dark
       | Ages.
        
       | DeathArrow wrote:
       | >Why are nuclear power construction costs so high?
       | 
       | Because there isn't enough will to make them cost less.
       | 
       | China is building lots of nuclear power plants.
        
       | w0mbat wrote:
       | If you think the construction costs are high, think about the
       | demolition cost when the facility reaches end of life, and is now
       | radioactive.
        
         | locallost wrote:
         | On top of that the cost of waste disposal is astronomic and in
         | most of the world without a permanent solution. And it is
         | usually not included in the actual calculation -- power plants
         | usually need to put some money on the side and into an index
         | fund, with the hope the fund eventually grows to be large
         | enough to cover the cost. But nobody really knows if it will
         | suffice, so it's likely the public will be on the hook. On top
         | of all the subsidies received during construction and
         | operation. Basically I view it as a type of graft.
        
       | DisjointedHunt wrote:
       | Way to bury the lede and avoiding the main point:
       | 
       | If you plan to build a Nuclear plant *TODAY*, there are financial
       | requirements and regulatory uncertainties that mean you're
       | sitting on high interest(and risky) loans/credit lines/asset
       | pledges etc that increase over time.
       | 
       | Very few banks or financial institutions are remotely interested
       | in setting up financing an endeavor that has an almost 0 chance
       | of success to completion since the 90s.
       | 
       | Environmental review has become a tool of environmental extremist
       | militants to derail and progress in energy. These organizations
       | are SO short sighted that they have been weaponizing the judicial
       | system against simple projects such as high voltage transmission
       | lines for the silliest of reasons which assures America that her
       | infrastructure will forever be stuck in the past.
       | 
       | The cost of operating a plant come after all this is considered.
       | All plants running today are roughly HALF A FUCKING CENTURY old.
       | What the US needs is easy access to cheap credit for people
       | willing to set up Nuclear plants.
       | 
       | We need incentive to invest in audacious increases in energy
       | output in exchange for meeting thresholds of performance. Right
       | now, you can kill yourself by filling out thousands of pages of
       | ridiculous review, hire some of the most expensive attorneys to
       | represent you in court to be granted to privilege of even having
       | basic clearance to START building while sitting on a fast
       | bleeding pool of credit, it makes NO sense.
        
         | fatcat500 wrote:
         | > Environmental review has become a tool of environmental
         | extremist militants to derail and progress in energy.
         | 
         | Hmm... I wonder why they are so bent on blocking the only
         | viable solution to climate change?
         | 
         | Almost as if they are being used a pawns to shift over the
         | control of energy to the government... after all, if I wanted
         | to nationalize every industry, I would start with the industry
         | upon which all other industries depend on: energy.
        
           | [deleted]
        
           | goodpoint wrote:
           | > they are being used a pawns to shift over the control of
           | energy to the government
           | 
           | ...by pushing for domestic solar panels? And improved
           | isolation, heat pumps, passive houses?
           | 
           | Sounds like the very opposite of centralizing energy
           | production.
           | 
           | So maybe we need a bit more evidence for your conspiracy
           | theory.
        
       | jason-phillips wrote:
       | Last year I did a fair amount of consulting work for the GAIN
       | initiative at Idaho National Lab [0].
       | 
       | They're doing so much good work with micro and modular reactors
       | that can basically be "dropped in" decommissioned coal-burning
       | sites because the infrastructure to tie into the electric grid
       | already exists.
       | 
       | It was expressed to me that selling this idea to the private-
       | sector energy industry was an uphill battle and uptake was very
       | slow to nonexistent.
       | 
       | [0] https://gain.inl.gov/SitePages/Home.aspx
        
         | hutzlibu wrote:
         | "selling this idea to the private-sector energy industry was an
         | uphill battle"
         | 
         | Could you elaborate on the why? I would asume, because of risk.
         | A solar plant, you can more or less just put anywhere, but a
         | nuclear power plant, even a small one, needs state permission,
         | has to meet extensive regulation, etc.
         | 
         | It would need some convincing for me, too, that a nuclear
         | reactor can be just a drop in replacement for coal. I would
         | think unknown risks, hidden costs due to regulations,
         | neverending building, etc.
         | 
         | And are we talking about a battle tested design, or is it new
         | technology? That sounds extra risky.
        
           | bozhark wrote:
           | Sounds like a good time to get into the energy sector.
           | 
           | Time for an HN Wind, water, solar, and nuclear company start-
           | up.
           | 
           | Who's in?
        
             | samstave wrote:
             | There was a nuclear startup that came through YC a few
             | years back... what happened to them?
        
               | pnw wrote:
               | Oklo is still around, recent news article wasn't great
               | though. https://news.ycombinator.com/item?id=29937836
        
             | _1tan wrote:
             | Here, email is avg@duck.com
        
             | cupofpython wrote:
             | I have government contracting experience in construction
             | management QAQC
        
           | 7952 wrote:
           | Not the parent but have some experience with this on the UK.
           | Typically energy companies will start with the technology
           | they want to develop and then find a site that will suit it.
           | Starting with the plot of land is the wrong way around. And
           | existing decomissioned sites are just an asset like any
           | other. They may be sold for a distribution centre or a data
           | centre. And the grid connection may be used by a new power
           | station built on a neighboring plot or to connect an offshore
           | wind farm.
        
         | samstave wrote:
         | That sounds like a brilliant idea. However, the tooling and
         | staffing requirements of a Coal Plant arent going to suffice
         | for a nuke.
         | 
         | How do they propose training for existing employees of a coal
         | plant.
         | 
         | We think of Coal folks as "dirty stupid miners from kentucky"
         | and we think of people that work at nuke sites as "white lab
         | coat wearing scientists"
         | 
         | The only dirty stupid person from Kentucky is Mitch.
        
           | MisterTea wrote:
           | > However, the tooling and staffing requirements of a Coal
           | Plant arent going to suffice for a nuke.
           | 
           | I am not sure what you mean by Tooling but I am sure these
           | retrofit reactors aren't going to be dropped off by UPS and
           | someone on site has to figure out how to plug it in.
        
           | SoftTalker wrote:
           | I think there is a good amount of overlap.
           | 
           | Operating the reactor is of course specialized. But once you
           | have the steam, the rest of the power plant is conventional.
           | Steam turbines, generators, and all the grid tie-in would be
           | mostly the same as a coal plant.
        
         | bozhark wrote:
         | We could always start _new_ companies...
        
         | cryptonector wrote:
         | Here's an idea: offer one of these micro/modular reactors for
         | _free_ to a developer of a new residential division, and
         | indemnify them by offering to remove it after at least N years
         | in operation and up to M years after that.
         | 
         | I would consider living in a division that has extra low-cost
         | electricity. It couldn't be zero cost because nuclear could
         | only provide base load power, unless one of these micro/modular
         | reactor types is so innovative that it can provide base _and_
         | peak load power, in which case such a division could have truly
         | zero-cost power for a bunch of years.
         | 
         | I.e., _promote_ the darned things, loss-lead if need be.
         | 
         | If the manufacturer won't take such risks, then might never
         | break through.
         | 
         | I bet that after a few years you could get such good press out
         | of it that other developers might sign on.
        
           | ethbr0 wrote:
           | > _Here 's an idea: offer one of these micro/modular reactors
           | for free to a developer of a new residential division..._
           | 
           | One word: Epcot
           | 
           | It's something Disney and Florida could probably agree on,
           | would be a wholesome PR story about investing in Walt's
           | futurism dreams, and would be easier to negotiate than with a
           | less-planned development.
           | 
           | And finally, what better showcase than "If it's safe enough
           | for Disney"?
           | 
           |  _Our Friend the Atom_ , indeed
           | https://m.youtube.com/watch?v=QRzl1wHc43I
        
         | photochemsyn wrote:
         | Construction costs are only part of the picture and utility
         | operators are well aware that they need to look at lifecycle
         | costs. This includes everything from costs of fuel rods (look
         | at the historically volatile uranium market), availability of
         | large volumes of cooling water (see more frequent droughts),
         | maintenance costs (maintenance being a major factor in the
         | retirement of California's nuclear power plants), security
         | costs, the cost of storing fuel rods onsite for decades, and
         | finally, decommissioning costs (as reactors themselves become
         | contaminated with in-situ activation products, i.e. radioactive
         | cobalt/iron/carbon/nickel isotopes).
         | 
         | It's basically a huge long-term liability that just doesn't
         | exist with solar/wind/storage, hydropower, or geothermal.
        
           | spoonjim wrote:
           | Can't you just dump radioactive uranium into the bottom of
           | the ocean? Can't imagine it would do much damage with all of
           | the water around it.
        
             | iancmceachern wrote:
             | No
             | 
             | https://theecologist.org/2009/mar/01/somalia-used-toxic-
             | dump...
        
               | Dylan16807 wrote:
               | That doesn't really show anything because it's such a
               | huge mixture and the vast majority is random toxic
               | chemicals, not plain old spent fuel.
        
               | stormbrew wrote:
               | I'm not saying I think dumping nuclear waste in the ocean
               | is a good idea (though I am also curious if there's been
               | specific impact studies done on it, if we're talking
               | about very very deep ocean) but I'm pretty sure somalia
               | isn't _in_ the ocean.
        
               | iancmceachern wrote:
               | They were dumping waste off the coast of Somalia, in the
               | ocean
        
               | Dylan16807 wrote:
               | "Bottom of the ocean" here doesn't mean on the
               | continental shelf.
        
             | JodieBenitez wrote:
             | There's life down there.
        
               | [deleted]
        
           | coredog64 wrote:
           | The largest nuclear plant in the United States (Palo Verde)
           | is in the middle of the Sonoran Desert and uses treated
           | sewage output for cooling. People probably won't stop
           | urinating during a drought.
        
             | cultofmetatron wrote:
             | oh thats cool. I imagine all the pathogens are effectively
             | killed for free
        
           | dqpb wrote:
           | > the cost of storing fuel rods onsite for decades
           | 
           | This environmental remediation cost is generally missing in
           | comparisons with all other energy sources.
        
             | iancmceachern wrote:
             | And mining too
        
             | anamexis wrote:
             | It's generally missing from the other energy sources as
             | well.
        
         | nradov wrote:
         | Private industry uptake is slow because the required changes to
         | staffing, security, and waste disposal are so expensive. Until
         | there are higher costs for CO2 emissions, it will be cheaper
         | for power companies to convert those facilities to natural gas,
         | or just shut them down.
        
           | Arrath wrote:
           | > ...waste disposal are so expensive. Until there are higher
           | costs for CO2 emissions...
           | 
           | Perhaps we should also examine the requirements and costs
           | associated with storing waste from coal-fired power plants.
           | Oft overlooked in favor of the fuel rod boogeyman.
           | 
           | https://www.nationalgeographic.com/environment/article/coal-.
           | ..
           | 
           | If we treated such waste with the care and security it
           | deserved, the cost equation may balance out differently.
        
             | krallja wrote:
             | One of the victims of coal was Pat McCrory in the 2016 NC
             | governors race. Signing "H.B. 2," the stupid transgender-
             | bathroom law, is often cited as the main reason for his
             | loss, but I believe the coal-ash spill[1] and subsequent
             | coverup scandal were decisive in his losing support in
             | rural areas.
             | 
             | I certainly agree: the true costs of coal should be better
             | believed!
             | 
             | https://en.m.wikipedia.org/wiki/Pat_McCrory#Duke_Energy
        
             | paulmd wrote:
             | Yes. The problem with nuclear is basically twofold:
             | 
             | first, the approval and regulatory process is deliberately
             | cumbersome and in obvious need of reform. Treating every
             | plant as a one-off design rather than standardizing has
             | enormously inflated costs. And generally much higher
             | scrutiny requirements for new designs have strangled the
             | ability to roll out better/safer designs. It's very similar
             | to what happens in the FAA with aircraft/engine designs, we
             | can design much better engines/reactors than we could in
             | 1960, but the new ones require an onerous approval process
             | while the old ones got grandfathered approval. So we only
             | build the old/worse ones!
             | 
             | If you don't want any more nuclear constructed, set policy
             | to that effect, don't use the approval process to
             | artificially inject costs to make it unfavorable. And the
             | disposal situation just needs to happen, period. The waste
             | has to go somewhere, we can't just have it sitting around
             | forever. Even if we never ran another nuclear plant ever,
             | the _existing_ waste still has to go somewhere, and that
             | process was dragged to a halt for political reasons too.
             | The Yucca Mountain repository needs to be moved forward
             | again.
             | 
             | Second, we need to stop letting coal externalize its costs.
             | Tax carbon emissions heavily, require secure disposal of
             | radioactive coal ash rather than letting it sit around in
             | storage ponds that eventually spill and destroy miles and
             | miles of land, etc.
             | 
             | Unfortunately, in both cases, the fossil fuel industry has
             | its finger on the pulse of washington and isn't going to
             | allow a trillion-dollar industry to be torn down without a
             | massive fight. Existing stakeholders are just too
             | entrenched for it to ever be successful and construction
             | costs/disposal/externalities are just the place where that
             | iceberg breaks the surface.
             | 
             | The "once you build the expertise, costs come down for the
             | Nth plant" is also probably true, but there's also other
             | things going on here with the process as a whole.
        
               | ncmncm wrote:
               | Doesn't seem to have helped any at Vogtle.
        
         | logifail wrote:
         | > micro and modular reactors
         | 
         | Umm, are these the ones that - relatively speaking - produce
         | significantly more radioactive waste than traditional designs?
         | 
         | "The next generation of small nuclear reactors will be big on
         | producing radioactive waste"[0]
         | 
         | "Small nuclear reactors produce '35x more waste' than big
         | plants"[1]
         | 
         | Sorry for the scepticism, but the nuclear lobby were the ones
         | that brought us the phrase "too cheap to meter"[2], and we're
         | still waiting...
         | 
         | [0] https://www.chemistryworld.com/news/the-next-generation-
         | of-n... [1]
         | https://www.theregister.com/2022/06/02/nuclear_reactors_wast...
         | [2] https://www.nrc.gov/docs/ML1613/ML16131A120.pdf
        
           | a_shovel wrote:
           | There was a discussion about this a few days ago, and this
           | comment thread [0] featured some criticism from experts on
           | these claims.
           | 
           | [0] https://news.ycombinator.com/item?id=31642424
        
             | logifail wrote:
             | and yet one of the comments from that thread stated:
             | 
             | > The correct answer is that there is no way to make
             | fission cost-competitive with the near-term price of
             | renewables + storage, or even with imported synthetic fuel
             | produced with renewables, waste or no waste. Thus, the
             | whole issue is moot. No such SMRs will be built except
             | where coerced funding carefully excludes true cost from the
             | process (as indeed happened on behalf of every single
             | utility reactor in operation
        
               | ClumsyPilot wrote:
               | There are countries where no combination of renewables +
               | storage is possible. Places with little sunlight and
               | little wind. What do they do?
        
               | logifail wrote:
               | Q: Do those countries already have long term nuclear
               | waste storage facilities?
               | 
               | There are countries that have been spectacularly failing
               | to provide their own such facilities for decades.
        
               | ClumsyPilot wrote:
               | Suppose they don't - so what is you proposed solution -
               | they live in the dark ages?
        
               | ncmncm wrote:
               | They today import fuel and burn it. They are equipped to
               | continue doing so.
               | 
               | They can add transmission lines, for cheaper importation.
               | 
               | As fuel synthesis -- ammonia and hydrogen -- comes online
               | and undercuts NG, they can import that instead, at times
               | when their transmission line power capacity or
               | availability is exceeded.
               | 
               | Fuel synthesis from solar in the tropics undercutting NG
               | extraction will be a big business. Synthesis using
               | reliable wind, in other places, likewise.
        
           | theptip wrote:
           | 35x of a small problem is not necessarily a big problem. You
           | need to quantify that objection. Nuclear waste can be
           | reprocessed (see Fast Breeder reactors); the concern raised
           | here is usually proliferation risk.
           | 
           | Given a binary choice, I'd rather have to deal with some
           | nuclear waste than climate change, and I don't even think
           | that climate change is an existential risk for humanity (just
           | likely to cause large and uneven/unjust levels of harm). I
           | think it's misleading to throw out individual objections like
           | this without considering the systemic trade-offs that we need
           | to make.
           | 
           | > the nuclear lobby were the ones that brought us the phrase
           | "too cheap to meter"
           | 
           | I think this kind of blame-throwing is unhelpful. I couldn't
           | care less what marketing claims were made in the past. Does
           | this technology make a good cost/benefit trade-off now, or
           | not? Specifically, compared to the other options we actually
           | have available to us now. That is the conversation I think we
           | should be having.
           | 
           | To address your object-level claim, as the OP describes in
           | detail, one reason nuclear is more expensive now is the ever-
           | increasing regulatory framework that has changed under the
           | feet of in-progress projects. Maybe those regulations have
           | resulted in a good ROI, but it's hard for me to buy that
           | claim. Nuclear is now 10-100x safer than most other
           | traditional/fossil forms of power generation[1], and it's
           | excruciatingly expensive to buy that safety margin. These are
           | tiny death rates caused by power generation.
           | 
           | The problem here is that it's political suicide to say "I
           | think we should reduce safety regulations to make nuclear
           | power 10x more dangerous, because that will avert far more
           | deaths from climate change". In practice perhaps a major push
           | for solar/wind/geothermal would be a higher-ROI solution to
           | our problems, but that's politically much harder to get
           | consensus on.
           | 
           | [1]: https://ourworldindata.org/safest-sources-of-energy
        
             | zackees wrote:
             | Post like this make me hopeful that rational minds will
             | prevail and we get clean abundant nuclear energy.
             | 
             | It's becoming clear that the climate alarmism isn't about
             | solutions or even a tax, but forcing us to purchase "carbon
             | credits" from Saudi Arabia and other sovereign wealth funds
             | ranked high in the ESG index.
        
               | ClumsyPilot wrote:
               | "It's becoming clear that the climate alarmism isn't
               | about solutions or even a tax, but forcing us to purchase
               | "carbon credits"
               | 
               | Every half educated climate activist knows thay carbon
               | credits are a fraud perpetuates by wallstreet types to
               | pretend they are doing something when they are not.
               | 
               | This post is breathtakingly uninformed. I suggest calling
               | it climate illiteracy
        
               | throw827474737 wrote:
               | Posts like this make me just despair... we couldn't even
               | scale up this to the world wide needs, but if we could
               | the waste would be huge.
               | 
               | Only thing that could get us out there and would only
               | make sense is going renewables everywhere in Manhatten-
               | like projects, but that won't happen because people
               | argueing like this got us into this situation where there
               | is allegedly only a binary choice between failed and
               | failed..
        
               | towaway15463 wrote:
               | > the waste would be huge
               | 
               | Isn't the opposite true? Nuclear fuel has the highest
               | energy density of any fuel source by a gigantic margin.
               | The entire US stockpile of waste since the 50s is only
               | 83,000 metric tonnes and could fit on a single football
               | field stacked less than 10 yards deep. If we could all
               | stop clutching our pearls about scary radiation and just
               | agree to store the spent fuel deep underground there'd be
               | absolutely no danger from it. Add to that the fact you
               | can recycle the fuel and get even more energy out of it.
        
               | logifail wrote:
               | > ESG
               | 
               | Is that the same ESG that Exxon is part of but Tesla
               | isn't?
        
             | throwaway23234 wrote:
             | > In practice perhaps a major push for
             | solar/wind/geothermal would be a higher-ROI solution to our
             | problems, but that's politically much harder to get
             | consensus on.
             | 
             | The most awesome thing you get from Solar is that it's
             | going to move forward no matter what. It's just something
             | you can do to lower your bill, or even disconnect from the
             | grid altogether. Personally I am off grid with no propane.
             | Almost unheard of in CA. Solar is now THAT cheap if you are
             | willing to put the up-front investment instead of paying
             | PGE $600 a month. And if you don't have the space - buy a
             | solar panel on a business from that other company they are
             | starting - it was on here a week or so ago.
        
               | theptip wrote:
               | I agree with the trend here. It's inevitable that solar
               | will displace fossil fuels eventually (especially when
               | you consider that fossil fuels will gradually increase in
               | price as the cost to extract goes up).
               | 
               | The problem is that moving to solar solely by riding out
               | the market-led transition won't happen fast enough. We
               | would need to subsidize a lot more to avert the worst
               | outcomes of climate change.
        
               | criley2 wrote:
               | Solar is a perfect solution for like 1/3 of the world for
               | like 1/3 of the day.
               | 
               | Batteries though, current batteries are a horrible
               | solution.
               | 
               | It's funny that this threads main objection to nuclear is
               | waste/mining/etc but the solar battery revolution
               | generates some far worse environmental effects getting
               | all those rare earth minerals. Compared to the tiny
               | amounts of fuel needed for reactors, the sheer amount of
               | metal needed for worldwide grid solar batteries is
               | astronomical.
        
               | ncmncm wrote:
               | Mentioning batteries can only distract from sensible
               | discussion, because only the tiniest fraction of utility-
               | scale storage will ever be batteries.
        
               | solardev wrote:
               | Solar is great at small scale, but at utility/grid scale,
               | that means needing to ramp up storage too. The grid
               | doesn't currently have much storage capacity (batteries,
               | pumped hydro, phase shifting materials, etc.) to support
               | current needs, much less future needs if we want to phase
               | out fossil fuels.
               | 
               | Getting the lithium and cobalt infrastructure up to scale
               | takes time and has a lot of geopolitical considerations.
               | It's not impossible but also far from trivial. It's not
               | just a matter of throwing up more panels and turbines and
               | calling it a day.
               | 
               | Unless we can solve storage at scale, generation at night
               | will continue to be an issue, and nuclear is the least
               | climatically damaging way to do that in the interim.
        
               | megaman821 wrote:
               | Isn't this the crux of the argument for investing more
               | money into solar/wind/storage vs nuclear. The solar and
               | wind stuff is already cheap enough, so its storage vs
               | nuclear. Will nuclear get cheaper faster than grid-scale
               | batteries? There is only so much capital that can be
               | invested, and potential advancements in batteries are
               | looking a lot more promising than advancements in
               | nuclear. I think the lion's share of money should be
               | going to renewables and batteries.
        
               | ncmncm wrote:
               | Batteries are the _most expensive_ storage. They cost per
               | kWh stored, while others cost mainly only per W inserted
               | or extracted. Only a minuscule fraction of utility scale
               | storage will ever be batteries, so even mentioning
               | batteries only details discussion.
        
             | SiempreViernes wrote:
             | > Does this technology make a good cost/benefit trade-off
             | now, or not? Specifically, compared to the other options we
             | actually have available to us now. That is the conversation
             | I think we should be having.
             | 
             | Then why are you making so many arguments about how we must
             | find ways to make it cheaper? That doesn't seem very
             | committed to the idea of discussing the _current_ cost
             | benefit trade-off.
        
               | theptip wrote:
               | The technology might make a good cost/benefit trade-off,
               | if we fixed the regulatory framework but not otherwise.
               | Or it might already make a good trade-off in the current
               | regulatory environment. I don't see the technology and
               | the regulatory framework as being tightly-coupled here,
               | though they do obviously affect each other. My point is
               | that we need to look at both of these factors, rather
               | than considering the regulatory framework as a given that
               | is set in stone, definitely correct, and something we
               | can't change.
               | 
               | In this case it may be easier to change the regulatory
               | environment than to come up with new dramatically-cheaper
               | nuclear technology that complies with the restrictions of
               | the current regulations (though that is happening too
               | with modular reactors).
        
               | SiempreViernes wrote:
               | > I don't see the technology and the regulatory framework
               | as being tightly-coupled here
               | 
               | You're saying the safety standards regulation is not
               | couple to the technology being used? Then what is it
               | regulating if it's not the technology?
        
               | avianlyric wrote:
               | > Then why are you making so many arguments about how we
               | must find ways to make it cheaper?
               | 
               | They don't appear to be making those arguments at all.
               | Just stating that Nuclear regulations sets safety
               | standards substantially higher than safety stands for
               | other power sources.
               | 
               | It quite reasonable to include a question about "how safe
               | is safe enough" when talking about trade offs. It's easy
               | to change safety standards, and they're constantly
               | evolving in all industries, so it's hardly disingenuous
               | to consider as part of the current cost benefit trade-
               | off.
               | 
               | However to insist that Nuclear energy can only be
               | evaluated in strict unchanging state, or to suggest that
               | GP was suggesting such an evaluation, is somewhat
               | disingenuous, and substantially undermines the
               | authenticity of your argument.
        
               | SiempreViernes wrote:
               | > Nuclear regulations sets safety standards substantially
               | higher than safety stands for other power sources.
               | 
               | Ok, but this surely only is an argument that _other_
               | sources should be regulated harsher (and fossil fuels
               | absolutely should), not that we must make nuclear less
               | safe.
        
               | logifail wrote:
               | > Just stating that Nuclear regulations sets safety
               | standards substantially higher than safety stands for
               | other power sources.
               | 
               | Errm, than renewables?
        
             | bipson wrote:
             | The whole problem with the whole argument is: It _isn 't_ a
             | binary choice.
        
               | theptip wrote:
               | I was using that phrasing to illustrate my ranking of
               | preference between those two options, I wasn't claiming
               | that we have a binary choice.
               | 
               | To be clear, I am explicitly advocating for a proper
               | cost/benefit analysis that keeps all options on the
               | table, rather than getting caught up on single factors
               | ("we can't do micro-nuclear because it produces more
               | nuclear waste"). As I mentioned later in the post, a
               | major push to renewables is another option (I also don't
               | see these as mutually exclusive).
               | 
               | I know that the most popular plan here is "push hard for
               | renewables". I like that plan; I think a Green New Deal
               | is an excellent idea. But empirically, how is that plan
               | going? If that's your plan A, do you think it's going
               | well enough to reject a plan B that you deem to be worse,
               | but still dramatically better than climate change? I
               | think the stakes are high enough that we should be
               | hedging our bets.
        
               | freemint wrote:
               | The number of plants you build is an integer valued
               | choice though. This has profound implications.
        
             | Octoth0rpe wrote:
             | > Nuclear waste can be reprocessed
             | 
             | Arguing that we should adopt smr reactors because their
             | proportionally larger amount of waste can be burned by a
             | much larger/more expensive reactor that we'd _also_ have to
             | fund/build? One that would likely cost >10b and take 20
             | years to construct? That doesn't seem like a good response
             | to the waste issues with SMRs.
        
             | logifail wrote:
             | > Does this technology make a good cost/benefit trade-off
             | now, or not?
             | 
             | Given the progress (or lack of it) with the EPRs[0], I
             | don't think "now" is something nuclear can claim to deliver
             | on.
             | 
             | If we'd started planning dozens of reactors a decade ago,
             | perhaps - but we didn't.
             | 
             | Nuclear simply isn't any kind of "quick fix", by a long
             | stretch.
             | 
             | [0] https://en.wikipedia.org/wiki/EPR_(nuclear_reactor)
        
       | photochemsyn wrote:
       | This article is _very_ light on the basics of nuclear fission and
       | could use some help there. Here 's a good source on background:
       | 
       | https://world-nuclear.org/information-library/nuclear-fuel-c...
       | 
       | To summarize, light water reactors rely on the production of
       | uranium fuel rods, which hold uranium enriched to about 3% U-235
       | relative to 97% U-238. Naturally occuring uranium ores are about
       | 0.7% U-235 and there is a large variation in the percentage of
       | uranium in a given ore by total rock mass, with a few deposits
       | being as much as 18% uranium ranging down to about 0.1%, which
       | most sources describe as the economically recoverable limit. This
       | will affect the cost of refueling a LWR (which has to be done
       | every ~3 three years).
       | 
       | Fission in LWRs is due to slow thermal neutrons, which can only
       | fission U-235 and Pu-239 (odd-numbered isotopes. Fast neutrons
       | are a different story, see above source for that.) These slow
       | neutron/U-235 events generate fission fragments (atomic masses in
       | the range ~80-110 and ~130-150 with peaks at 95 and 135), gamma
       | rays, and free neutrons. The initial energy distribution for
       | heating the circulating fluid (water) is about 85% fission
       | fragment kinetic energy, and about 15% gamma ray and neutron
       | kinetic energy.
       | 
       | Some of the neutrons are captured by U-239, forming plutonium-239
       | (and other transuranics) - which is also subject to fission, and
       | over the lifetime of the fuel, about 66% of this formed Pu-239 is
       | itself fissioned, adding to the total energy output.
       | 
       | However, the actual heat produced by the reactor is also due to
       | long-term decay of the fission fragments inside the fuel rods
       | (about 6% of the total). This latter 6% is important because even
       | if you halt the initial fission process, the reactor won't just
       | go to zero, it still has to be cooled to prevent overheating and
       | meltdown, as do the 'spent' fuel rods. It takes about ten years
       | for used fuel rods to go from 10 kW decay heat/ton to 1 kW decay
       | heat/ton. Storage of used fuel rods adds significantly to the
       | operational costs of the reactor over time.
       | 
       | Long-term storage of used fuel is necessary almost entirely
       | because of the transuranics, which are alpha-radiation emitters
       | with half-lives of thousands of years. Most of the fission
       | fragments appear to decay via faster beta/gamma processes.
       | 
       | Finally, there are the activation products to consider. Tritium
       | is formed in the primary circulating water coolant loop, and is
       | highly radioactive which is why this loop has to be isolated and
       | its heat transferred to the secondary coolant loop which drives
       | the steam turbines. Note here that reactors have to use a lot of
       | water, at least as much as a coal-fired power plant, and these
       | systems need to be highy engineered to prevent breakdowns, which
       | would lead to meltdowns (Fukushima failure mode). The other
       | activation products form in the reactor itself - carbon-14,
       | cobalt-60, iron-55, nickel-63. This significantly adds to the
       | cost of nuclear reactor decommissioning as the entire reactor
       | body has to be treated as high-level waste.
       | 
       | These factors explain why nuclear reactors have to be
       | overengineered relative to traditional fossil fuel power plants,
       | oil refineries, etc. which regularly suffer major accidents and
       | fires - but those accidents don't lead to 100-year+ exclusion
       | zones around the accident sites, so it's deemed acceptable. Not
       | to belabor the point, but wind/solar/storage also has much lower
       | costs for these reasons. Security vis-a-vis terrorism,
       | cyberattack, military conflict, etc. is also a major related
       | cost.
        
       | TheDudeMan wrote:
       | Because reactors are much too large, causing far too slow of a
       | design iteration cycle and inability to leverage economies of
       | scale and mass production.
        
       | jokoon wrote:
       | I'm not an expert but I'm a bit skeptical about small nuclear
       | reactor designs.
       | 
       | I want to believe they would be cost effective, but... It doesn't
       | seem there is a prototype that is cheap?
        
         | larsrc wrote:
         | Prototypes are never cheap. Prototypes are where all the
         | research and design costs go. Once you have a design that works
         | and doesn't have to be customised for each instance, you can
         | get economics of scale because now you're just producing the
         | same items over and over.
        
       | daniel-cussen wrote:
       | Masochism in part, and secondly the Simpsons blackballing the
       | entire industry. Nobody wanted to become a nuclear engineer after
       | that shitty show. The creators of the Simpsons are nuclear war
       | surrender monkeys.
       | 
       | Context, in the show, the groundskeeper which is a discriminatory
       | stereotype of Scots, says the French are "cheese-eating surrender
       | monkeys", another discriminatory stereotype, but it's fine
       | according to the bitchvictim media's rules because since they're
       | both white it counts as "poking fun" and not "bigotry". Tucker
       | Max said "cheese-eating surrender monkey!" to a French girl at a
       | bar, game over right there even for a guy with crazy game like
       | him, because it's fucking insulting. Literally calling them
       | monkeys AND submissive cowards? Well if the Simpsons can do it to
       | others, I can do it to the Simpsons. One of my surnames is
       | French, I'm part French, my great-grandfather spoke French and
       | was a Francophile following the Blitzkreig with a map hoping the
       | French could turn the invasion around, he was rooting for them.
       | It's just not fucking funny. The Simpsons is a bigoted show,
       | don't think you can repeat any of those jokes. So this is what I
       | get to reply to the Simpsons, and all those losers: The Simpsons
       | are nuclear armageddon surrender monkeys. Whole bitchvictim media
       | with them.
       | 
       | Going back to the topic of nuclear engineers, with 22 minutes of
       | slander on television on every day specifically against them
       | that's the hottest show on television decade after decade, like
       | only a son of a nuke would become a nuke.
       | 
       | So that's also sabotage, then the public is like a thousand times
       | as sensitive to a nuclear accident than to a coal plant shitting
       | into the air we breathe.
       | 
       | https://news.ycombinator.com/item?id=31598892
       | 
       | https://hn.algolia.com/?dateRange=all&page=0&prefix=false&qu...
        
       | dqpb wrote:
       | I would like to see a comparison of energy sources that includes
       | environmental remediation costs.
        
       | memco wrote:
       | There are efforts in our area to expand solar and they are
       | offering programs to the public to subsidize some of the
       | construction with shares of the panels. Customers are advised
       | that they will not make money but they'll get back some
       | percentage of their shares each month. Additionally the power
       | company offers a renewable power option for which they charge
       | extra over the standard rate. Given this: will we see a nuclear
       | option to help subsidize the cost? I would love the cost to come
       | down but I feel like they're never going to come down if we don't
       | build it anywhere. Would love to help get it going although I
       | have nowhere near enough money to make an offer other than to be
       | a willing paying customer to whoever can get service to me.
        
       | dest wrote:
       | On this topic, see the excellent website
       | https://whatisnuclear.com/ and its webpage about economics
       | https://whatisnuclear.com/economics.html
        
       | xroche wrote:
       | > Because nuclear plants are expensive, and they take a long time
       | to build, financing their construction can also be a significant
       | fraction of their cost, typically around 15-20% of the cost of
       | the plant. For plants that have severe construction delays and/or
       | have high financing costs (such as the Vogtle 3 and 4 plants in
       | Georgia), this can be 50% of the cost or more.
       | 
       | This is why nuclear power plants should be state-sponsored
       | projects. States typically have loans at 0% rate, or even
       | negative interests.
        
         | stewbrew wrote:
         | The state better invests the money in cheaper technologies. Why
         | should it waste the money on nuclear plants?
         | 
         | Edit: To the downvoters: Seriously, why should the state waste
         | money on a technology that doesn't work and never has? More
         | than half of France's nuclear power plants are currently
         | offline and in maintainence mode. Maybe also because they could
         | not produce electricity at market prices.
        
           | ryan93 wrote:
           | They get 80% of their electricity from nuclear. And even
           | export some.
        
         | pmyteh wrote:
         | Yes, probably. This can bring its own problems, though. In
         | Britain, for example, the Treasury is extremely reluctant to
         | approve new public capital projects, even at negative real
         | interest rates. Investment is effectively rationed by requiring
         | a benefit/cost ratio of above 2 (at net present value, after
         | making heavy optimism bias adjustments) before it will approve
         | funding. New nuclear won't get close to that on any
         | conventional appraisal, which is one reason why _our_ current
         | nuclear new build is happening on an eye-wateringly expensive
         | private finance arrangement.
        
           | ImHereToVote wrote:
           | This is because of "not corruption"
        
             | pmyteh wrote:
             | It's mostly because the ability of the economy to produce
             | positive economic (but not direct financial) returns on
             | very cheap credit is basically infinite, and borrowing
             | money to invest in all of these things would need a huge
             | amount of extra tax revenue to pay off the loans, which is
             | hard. (Even in principle it's not straightforward for
             | government to capture the consumer surplus of
             | infrastructure investment. And in practice tax increases
             | are politically problematic). So it's rationed, instead.
        
         | strainer wrote:
         | A project should garner favorable financing arrangements for
         | its merits, not for its risks.
        
           | danans wrote:
           | Merit must be determined in a way that includes known risks.
           | Anything otherwise would be fraudulent.
        
         | Krasnol wrote:
         | Why should states invest it's taxpayer money into overpriced
         | and slow technology when there are cheaper and fast improving
         | alternatives?
        
           | BurningFrog wrote:
           | Note that solar and wind power are NOT alternatives to the
           | predictable base load of nuclear, hydro, or fuel burning
           | power generation.
        
             | Krasnol wrote:
             | "Steve Holliday, CEO National Grid: "The idea of large
             | power stations for baseload is outdated""
             | 
             | https://energypost.eu/interview-steve-holliday-ceo-
             | national-...
        
               | trashtester wrote:
               | This is a sales pitch from someone who makes a living
               | from selling grid capacity.
        
               | jacquesm wrote:
               | But is it true or not?
        
               | trashtester wrote:
               | I'm not sure he's lying on purpose. But it's pretty
               | common that people just stop thinking further when
               | someone tells him what they want to hear.
               | 
               | The article certainly dismisses the need for storage way
               | too easily, imo. It claims that consumption can be
               | adjusted to match supply. There are not that many uses of
               | electricity where you can simply lower your consumption
               | when the supply is low.
               | 
               | There are some cases, like car batteries that can, sort
               | of, be seen as consumption, but unless your car has some
               | extreme storage capacity, you typically want to be able
               | to recharge it when YOU need to have that range, instead
               | of when the power company has additional supply.
               | 
               | And if you don't want to use fossil fuels for heating,
               | the power saved by not charging your car is NOT enough to
               | keep your house warm for a few cold days with no winds
               | (unless you live in a place with no real winter).
        
           | xienze wrote:
           | Because transitioning over to green technology is a decades-
           | long project, it's not something you can just snap your
           | fingers and make happen, as many countries are discovering.
           | You still need non-renewable energy sources to fill in the
           | gaps that renewables currently have.
        
             | Krasnol wrote:
             | You don't have to transition completely in decades. You can
             | start today. Meanwhile building a single nuclear reactor is
             | a "decade-long" project, you can't start today and in the
             | end you're still left with an old and expensive tech while
             | the green tech moved ahead rapidly during the same time.
             | 
             | Germany managed to replace almost half of their generation
             | in 2 decades:
             | https://www.cleanenergywire.org/news/germanys-renewable-
             | powe...
             | 
             | When they started out the technology was terrible and
             | managed to do all that despite a just recently retired
             | government which did everything to stop further expansion.
        
               | trashtester wrote:
               | > Germany managed to replace almost half of their
               | generation in 2 decades:
               | https://www.cleanenergywire.org/news/germanys-renewable-
               | powe...
               | 
               | Still, only 16% of Germany's total energy consumption
               | comes from renewables:
               | 
               | https://www.cleanenergywire.org/sites/default/files/style
               | s/g...
               | 
               | If Germany wants to replace all uses of fossil fuels for
               | heating, transportation, industrial use, etc, with
               | renewables, HUGE investments remain.
               | 
               | In particular, giving up the ability to smooth out
               | variations in production without the use of fossil fuels
               | will be extremely costly, unless the cost comes down by
               | at least a factor or 50.
        
               | xienze wrote:
               | > You don't have to transition completely in decades. You
               | can start today.
               | 
               | Yes, I agree. If you start today, you'll be done in
               | decades. The boneheaded move is to start a green energy
               | transition and immediately start decommissioning existing
               | nuclear power plants and stonewall creating new ones by
               | throwing up your hands and saying "well it'll take
               | forever to build them." By the way, have you ever
               | considered why it takes so long to build nuclear power
               | plants? It's a political and environmental special
               | interest problem, not a technical one.
               | 
               | At the end of the day, when the wind isn't blowing or the
               | sun isn't shining, you still have to generate power
               | somehow. Until the day that problem is solved (that's the
               | "decades" part), you want something like nuclear power to
               | fall back on.
        
               | Krasnol wrote:
               | I don't know why you're ignoring my reality example.
               | Germany is part of a EU wide market and it just works.
               | Also it's not like you put all your wind on one spot.
               | There is always wind somewhere for example.
               | 
               | The idea that it's an "political, environmental and
               | special interest" problem while we're watching several
               | nuclear plants being FAR over budget and over due being
               | constructed in pro-nuclear countries proves that your
               | argument is false.
               | 
               | So basically: everything you wrote there is wrong...why
               | are you doing this?
        
               | trashtester wrote:
               | > There is always wind somewhere for example.
               | 
               | There is always wind somewhere. But grid capacity is not
               | free, in fact it is quite expensive. Let's say, on a
               | given day, the only place in Europe with reasonable winds
               | would be west of Cadiz, transporting all that power
               | through Spain, Portugal and France to cover the needs of
               | all of Europe, would require immensive grid capacity
               | expansion. And even with super-high-voltage, the losses
               | before the power reaches Estonia would be huge.
               | 
               | Also, if this load causes a brownout in Spain, due to
               | improper maintaince, for instance, all of Europe could go
               | dark, cold and stop moving (in a time after fossil
               | fuels).
               | 
               | (I can imagine seeing this from space during some cold
               | winter night around 2045, all the lights in Western and
               | Central Europe disappear at once. Only Norway and parts
               | of Sweden can be seen, since they have their hydro
               | power.)
               | 
               | In other words, while a better grid can mitigate _some_
               | of the variability of renewable supply, you still need
               | massive expansion of storage capacity when you stop using
               | natural gas, especially when you switch heating and
               | transportation to use electricity too.
               | 
               | Seen from the outside, it surely looks like the German
               | population has been seriously misled.
        
             | petre wrote:
             | Yeah, we've seen how it (hasn't) worked out for Germany.
        
           | xyzzyz wrote:
           | There are no cheaper and fast improving alternatives to
           | provide plentiful electricity at 9pm every single day.
        
             | Kon5ole wrote:
             | Here are a few:
             | 
             | https://www.energy-storage.news/department-of-energy-
             | confirm...
             | 
             | https://www.energy-storage.news/batteries-at-worlds-
             | largest-...
             | 
             | https://www.energy-storage.news/energy-dome-launches-4mwh-
             | de...
        
               | xyzzyz wrote:
               | That's not cheaper. Run the numbers and you'll see. Solar
               | by itself is indeed pretty cheap per kWh if you don't
               | care about matching supply with demand, but storage very
               | much is not. If it was, you'd see investors build
               | standalone storage, to buy cheap electricity, store it,
               | and resell when demand goes up. This is not what's
               | happening: instead, existing projects are based either on
               | heavy government subsidies, or on vanity buyers, who want
               | to pay above market prices to signal eco awareness, like
               | Starbucks in one of your links.
        
               | Kon5ole wrote:
               | The problem with running the numbers is that the actual
               | numbers for nuclear are basically unknowable and most
               | governments have given a taxpayer insurance that covers
               | this unknown number "in blanco".
               | 
               | This means that most of the costs that will be caused by
               | operating a nuclear power plant are not included in the
               | costs of operations, and therefore not in the "price per
               | MWh" or similar numbers. We don't know what this number
               | is but we do know it's a very large number, and by
               | removing it from the resposibility of the plant operators
               | it represents a very large hidden subsidy for nuclear
               | power.
               | 
               | Chernobyl and Fukushima are the familiar elephants in
               | this particular room of course with he most recent
               | estimate for Chernobyl passing 600bn usd in 2016 (and
               | counting still of course and for the forseeable future)
               | but I like to use the Asse II salt mine in Germany as a
               | more digestable example.
               | 
               | This mine was used to store nuclear waste in the 70s
               | which turned out to be a very bad mistake that has to be
               | fixed in the coming few decades. The cost of this
               | project, (estimated to be at least several bn euros) is
               | not added to the cost of nuclear, it's just charged to
               | the current taxpayers. The power plants that generated
               | the waste stored in this mine are closed long ago but
               | they keep costing money decades later.
               | 
               | Nuclear seems cheap because we're paying for it with
               | credit cards issued to our grandchildren.
        
           | sofixa wrote:
           | Because you can start a project with the existing, proven and
           | expensive tech _today_. Grid-scale storage is purely
           | theoretical today (bar pumped-up hydro, which is infeasible
           | in most locations). There 's lots of hope, and money should
           | be invested in the various alternatives, absolutely. But
           | nobody can say when and if that tech would be ready.
        
             | jeffbee wrote:
             | This is really not correct at all. Half of the utility-
             | scale generating projects waiting for interconnect approval
             | are combined solar and battery installation. Grid-scale
             | storage is a solved problem, technically and economically.
             | There were over 400GW of grid storage project proposed in
             | the U.S. at the end of 2021.
        
             | renewiltord wrote:
             | According to this commenter, we can't build with the proven
             | and expensive tech today because we don't have it any more
             | 
             | https://news.ycombinator.com/item?id=31682876
        
               | sofixa wrote:
               | Depends who.
               | 
               | For EPR, we're almost over the hump, with the latest
               | projects in Finland and France coming online and becoming
               | fully operational in the near future.
               | 
               | Rosatom has continued pumping out reactors at a decent
               | rate.
        
               | [deleted]
        
               | renewiltord wrote:
               | Looks like they should do it and we shouldn't in America
               | unless we can somehow allow them to build ours (I think a
               | fear of competition will make this infeasible but if
               | we're lucky we'll get the stuff).
        
             | orthecreedence wrote:
             | Storage is not not purely theoretical!! In fact there are
             | <lists five completely theoretical storage methods that
             | rely on stable climate or perfect geography>.
        
             | epistasis wrote:
             | This is inaccurate, GW scale batteries could be deployed
             | today, but because storage is so scalable it's often a
             | better idea to build multiple smaller batteries that help
             | alleviate grid congestion.
        
               | belorn wrote:
               | A month or so back someone posted a report by a financial
               | investing advisor for the energy sector, and they were
               | pretty clear what is and what isn't economical viable
               | right now.
               | 
               | Solar + storage of 1-6 hrs can be made economical viable
               | as long as the storage can have 365 discharge cycles each
               | year, assuming prices get high enough each such cycle.
               | Each unit of storage get a return on investment each day,
               | and each are used fully at the point in time when the
               | market price is at peak.
               | 
               | Under those precise circumstances the economics of
               | storage is cheaper than nuclear. The only other cheaper
               | alternative to nuclear is to use renewables when the
               | weather is optimal and fossil fuel when the weather is
               | not optimal, or just use fossil fuels (through that is
               | just a waste of money and the climate).
               | 
               | Naturally this advisor firm could be wrong and someone
               | here could start the world first economical viable
               | operation that uses wind for renewables and then charge a
               | reverse hydro operation. It would make for a nice news
               | item.
        
               | epistasis wrote:
               | Making such broad statements about economical versus not
               | economical is difficult, because batteries serve so many
               | purposes and have so many revenue streams that deployment
               | is highly locational, depending on the specifics of the
               | grid and where and when demand causes congestion.
               | 
               | There's also little incentive to install storage when
               | solar and wind penetration is low, but as higher
               | percentages of the grid is powered by renewables, then
               | storage quickly becomes far more attractive.
               | 
               | Currently, there are 14.5GW of batteries in development
               | across the US, and this is just a tiny nascent industry.
               | Even as a small industry, this is many times the power
               | capacity of nuclear currently in development.
               | 
               | This biggest challenge with batteries right now is low
               | supply, and competing with demand from EV production,
               | which provides higher margins:
               | 
               | https://www.reuters.com/business/sustainable-
               | business/how-ba...
        
               | belorn wrote:
               | If we are talking about the US and not like places like
               | northern Europe, then they have a lot of existing
               | capacity for fossil fuel production. The cheapest way to
               | produce energy would be to just add more renewables and
               | use that fossil fuel whenever that weather isn't optimal.
               | Batteries might be competitive to fossil fuel in places
               | such situation as highlighted by the financial advisor,
               | ie when they can discharge fully each day of the year at
               | the maximum price point.
        
               | epistasis wrote:
               | The batteries can often be cheaper than fossil fuels,
               | especially when colocated with existing solar. Most solar
               | designs currently under size the inverters compared to
               | maximum solar power output, to get the cost optimal
               | balance. Batteries on-site allow storage of that extra DC
               | energy, and then reuse of the same inverters outside
               | normal solar generation hours to discharge the batteries.
               | 
               | This means that hitting the cost peak is really easy for
               | batteries.
               | 
               | As this cheapest form of energy begins to dominate, and
               | the "baseload" generators like coal or combined cycle gas
               | become more expensive than solar, then it becomes less
               | economical to run the "baseload generators because they
               | don't have sufficient price support during the peak solar
               | output times. This will raise the night time prices of
               | energy, as the daytime prices decrease, and eventually
               | storage plus solar becomes cheaper than new "baseload"
               | facilities, and then cheaper than continuing to run
               | existing "baseload" facilities.
               | 
               | I put "baseload" in quotes because on the past baseload
               | meant cheapest energy, in addition to slow and expensive
               | dispatchability. That is all changing.
        
               | belorn wrote:
               | > and then reuse of the same inverters outside normal
               | solar generation hours to discharge the batteries.
               | 
               | Yes, if we are talking about hours of capacity then
               | batteries can be very cost competitive to fossil fuels.
               | That is exactly what the financial advisor stated in
               | their report.
               | 
               | In areas where solar + batteries can reliable handle all
               | year round demands for energy, those technologies should
               | just replace fossil fuels. There will likely be some
               | natural gas plants that get subsidies to exist as reserve
               | in case there is a sudden weather change, but nuclear
               | wouldn't be a great option in such places.
        
               | jacquesm wrote:
               | A big variable too is the price development of batteries,
               | which is trending in the right direction due to more and
               | more production capacity coming on-line but once grid
               | storage and EVs start to compete for those batteries the
               | price could well be going up.
        
             | pydry wrote:
             | >Grid-scale storage is purely theoretical today bar pumped-
             | up hydro, which is infeasible in most locations
             | 
             | Viable locations arent in short supply at all:
             | 
             | https://www.sciencealert.com/scientists-
             | spot-530-000-potenti...
        
               | sofixa wrote:
               | They aren't in short supply, but aren't present
               | everywhere - e.g. in Europe there's nothing north of
               | Slovakia. If Denmark wants storage, they have to work
               | with other countries and rely on transit. It's even worse
               | for the Baltics.
        
               | arethuza wrote:
               | Norway, UK and Ireland aren't in Europe?
               | 
               | Edit: I was a bit puzzled as there are already pumped-
               | storage plants in Wales and Scotland with more planned.
        
               | raphaelj wrote:
               | Well, that also applies if Denmark wants uranium, oil or
               | gas.
        
               | freemint wrote:
               | You know that you can just pump all the water Denmark
               | has. Build underground caverns full with air, let water
               | in for energy and pump it out later.
        
               | Krasnol wrote:
               | You make it sound like it's some burden while the EU grid
               | is actually a single market with a significant expansion
               | last year: https://www.tennet.eu/our-grid/international-
               | connections/nor...
        
             | Krasnol wrote:
             | You make it sound like it's some SciFi tech where in
             | reality Germany has replaced half of it's whole production
             | with true green energy in the last two decades. Replacing
             | nuclear years ago.
        
               | larsrc wrote:
               | That's inaccurate. Germany has replaced half of its
               | _electricity_ production with renewables (modulo
               | dispatchability), but electricity only accounts for a
               | quarter of the energy usage. We frequently have to import
               | electricity from France now, where it's largely made by
               | nuclear, and electricity in France is a lot cheaper than
               | here. Closing the German nuclear plants was grand scale
               | stupid.
        
           | babypuncher wrote:
           | Hedging your bets. We know nuclear works. Grid-scale energy
           | storage for renewables still feels far fetched. Maybe it's
           | not. Either way, we should not put all our eggs in one
           | basket.
        
             | 7952 wrote:
             | We shouldn't put all our eggs in one basket. But I don't
             | think rapid growth of the battery industry is far fetched
             | at all. It has already experienced massive growth and is
             | ridiculously mass producable. And we will need batteries
             | anyway for electric cars.
        
               | trashtester wrote:
               | The battery capacity needed to replace all cars with
               | electric ones is about two orders of magnitude lower than
               | the battery capacity needed to replace all fossil fuels
               | with wind and solar, at least in temperate regions, where
               | you need heating during the winter.
               | 
               | According to this MIT study, the cost (LCOE) of doing
               | this today, would be $3000/MWH:
               | 
               | https://www.greencarcongress.com/2021/08/20210829-mitei.h
               | tml
               | 
               | Even if the cost of batteries continue to come down by x4
               | in price every decade from now on, it will take 30-40
               | years for prices to come below current energy prices.
               | 
               | If we hit an S-curve before then, it could take much
               | longer.
        
             | jacquesm wrote:
             | No, it also works:
             | 
             | https://arstechnica.com/information-
             | technology/2018/04/austr...
             | 
             | We just need a lot more of it, but it _definitely_ works.
        
         | fulafel wrote:
         | States tend to have self imposed debt limiting policies, so
         | opportunity costs are still there for investments.
        
         | Tenoke wrote:
         | If you think 50% cost overruns and overheads are uncommon for
         | state sponsored projects.. you will be right but only because
         | often the actual numbers are much higher.
        
         | sfe22 wrote:
         | Zero percent or negative just means people involuntary pay by
         | inflation. There is not free lunch (someone had to work to make
         | it)
        
           | imtringued wrote:
           | If there is a negative interest rate of 4% on cash, then the
           | easiest way to avoid it would be to lend out your money at
           | 0%. Since there is no growth dependence and excessive savings
           | do not grow automatically anymore there is no need for
           | inflation and the central bank can do price level targeting
           | instead.
        
           | kube-system wrote:
           | No, interest rates are quoted nominally -- i.e. they are
           | independent of inflation.
           | 
           | If you purchase a negative interest rate instrument and
           | experience inflation, you will lose real value to both.
        
           | trashtester wrote:
           | It is a tax on deposits. You only have to pay if you have
           | deposits. Basically, it means the saver has to pay to store
           | value as currency. On the other hand, they also have to pay
           | if they want to store other valueables, such as gold or the
           | most ancient store of value of all, grain.
           | 
           | I don't think a negative real interest rate is inherantly
           | unfair, any more than it was unfair to have 10% of grain go
           | to waste 3000 years ago when storing for a bad year.
           | 
           | If you had 7 good years, and expect 7 bad years, the utility
           | of the stored grain may be way higher in the bad years than
           | the good years, even when accounting for the waste. The same
           | goes for cash.
           | 
           | The same can be true with cash.
           | 
           | To demand that cash maintains its purchasing power, is the
           | same as ancient farmers demanding to purchase grain from
           | their neighbour (who did save) in a bad year as they
           | themselves got paid for their grain during the good years.
           | 
           | In periods of growth, we may start to think that positive
           | time preference is natural. But the fact is that throughout
           | most of human history, we would switch to negative time
           | preference in good times, since we expected bad times to come
           | back. During good times, humans would store grain, dry meat,
           | fish and fruit, build housing, tools or boats, all of which
           | were investments into goods that were likely to gradually
           | perish over time.
           | 
           | Even after people started to use coins, this was true. If you
           | produced a surplus during one year, you could trade it for
           | cold coin instead of storing it. But not only was there a
           | risk that the coins would be stolen or otherwise vanish, it
           | was also highly likely that at the time where you needed to
           | spend that goal, prices would be higher.
        
             | imtringued wrote:
             | One has to consider that the storage capacity of the
             | economy isn't infinite. Charging money for storing
             | something is a very straightforward business model. When
             | your economy is growing the storage capacity appears
             | endless as no storage is actually needed, you can just
             | produce the good in the future with your expanded
             | production capacity. Once the economy stops growing for
             | even a single year, then you will effectively hit the
             | storage capacity of the economy and must pay to store
             | additional goods that are intended to be consumed in the
             | future.
        
           | danuker wrote:
           | Negative interest rates mean more than your units of currency
           | going down in purchasing power.
           | 
           | It also means them going down in number (a bank CHARGES you
           | for the costs incurred holding your money).
        
         | this_user wrote:
         | > This is why nuclear power plants should be state-sponsored
         | projects. States typically have loans at 0% rate, or even
         | negative interests.
         | 
         | Not anymore with inflation at around 8% in many western
         | countries. And even so, construction and operation still remain
         | expensive. France's EDL is basically bankrupt if it were not
         | for the state backstopping them. However you want to slice it,
         | nuclear power is not economically viable, and looks even worse
         | compared to the rapidly decreasing costs of renewable sources
         | of energy.
         | 
         | So why waste more money on an obsolete technology rather than
         | use it for solving the remaining issues with renewables like
         | energy storage? Invest that money in battery technology and
         | everything that comes with that. That approach will do a lot
         | more good in the long run than trying to keep the nuclear
         | industry on life support even though it has failed for half a
         | century to deliver on its promises.
        
           | samstave wrote:
           | Inflation is WAY fn higher than 8%.
           | 
           | Try more like 30%
           | 
           | Go to costco's meat section, walmarts juice section, any fn
           | gas station.
           | 
           | Fleecing is whats happening.
           | 
           | EDIT:
           | 
           | Cool, clearly you arent tracking prices like I do.
           | 
           | Costco's meat section is up ~22%$
           | 
           | Walmarts Juices are up ~30%
           | 
           | We all know what gas prices are. $7 a gallon in Napa.
        
           | outworlder wrote:
           | > So why waste more money on an obsolete technology
           | 
           | It is not obsolete. At all. You can argue that some reactor
           | designs should not be used and I would agree. But fission is
           | the only answer we currently have for baseline power that
           | doesn't involve burning things. It will become obsolete if we
           | can ever make fusion work.
           | 
           | We should be deploying more reactors. There are small
           | reactors (shipping container-sized) that could be used to
           | power small towns and are pretty safe. Good luck getting one
           | approved and installed in your neighborhood. It's not the
           | tech that's being held back, it's people.
           | 
           | Look, I love batteries, I drive EVs since 2015. But if we
           | want to avoid the worst effects of climate change, we need to
           | provide cheap and reliable baseline power 24/7. There's not
           | enough time to do so with batteries alone.
        
             | derriz wrote:
             | The electricity from small modular reactors is far more
             | expensive than that from large reactors - about twice as
             | expensive by some estimates[1]. They also produce more
             | waste per MWh generated.
             | 
             | The industry has been pushing in the opposite direction
             | with larger reactors like the EPR[2] to reduce costs.
             | 
             | When measured by LCOE, a MWh from a new conventionally
             | sized nuclear plant is 4 to 7 times as expensive as a MWh
             | from solar PV, then SMR are simply out of the question from
             | a cost point of view.
             | 
             | [1] https://publications.csiro.au/publications/publication/
             | PIcsi... [2]
             | https://en.wikipedia.org/wiki/EPR_(nuclear_reactor)
        
               | trashtester wrote:
               | Solar is great in deserts, where the sun always shines
               | and were the main use of electricity is air conditioning
               | during the hottest parts of the day.
               | 
               | Solar is near-useless in colder areas, where you want to
               | use the power for heating in the winter.
               | 
               | If you include the storage + grid expansion needed to
               | compensate for the intermittent nature of most renewables
               | (especially if you don't want to rely on fossil fuels
               | when the wind is not blowing), the LCOE of many of them
               | will be many times higher than just the production cost.
               | 
               | Meanwhile, Korea claims to be able to construct Nuclear
               | capacity at prices down to $0.03/kwh with their APR1400
               | reactors:
               | 
               | https://www.kns.org/files/pre_paper/34/15A-435%EC%9D%B4%E
               | A%B...
               | 
               | That's at least an order of magnitude lower than the cost
               | of renewables when constant output is required.
        
               | Gwypaas wrote:
               | Awesome then that on-shore wind is even cheaper than
               | solar, and exists at night. Then for some more, still
               | less than 1/3 to 1/4 of nuclear you get off-shore wind
               | with higher capacity factors.
        
               | trashtester wrote:
               | When the wind is not blowing, the cost of wind per kwh is
               | infinite.
        
               | freemint wrote:
               | > Solar is near-useless in colder areas, where you want
               | to use the power for heating in the winter.
               | 
               | Heating is super awesome with renewables. As you can
               | store heat in an well isolated home. Yes you need
               | capacity for that but heat pumps well a lot with that.
        
               | trashtester wrote:
               | > Heating is super awesome with renewables. As you can
               | store heat in an well isolated home.
               | 
               | Sounds like you're not speaking from experience.
               | Actually, houses are pretty lousy batteries. Most people
               | have a range of only a few degrees that they find
               | comfortable indoors. They will tend to set the thermostat
               | to about the middle of that range. If they turn off the
               | head, the temperature will go the lower end of that range
               | after a few hours. Very few hours if it's really cold
               | outside, and that's when it matters most.
               | 
               | Admittedly, my house is old and not super-well isolated,
               | but during the coldest days of winter (around -20C), it
               | can easily require 10kw, constantly, to keep it warm
               | enough to prevent my wife from becoming agressive.
               | 
               | If we turn off the power for 2 hours, it's already pretty
               | cold.
               | 
               | Heat pumps would reduce overall energy consumption, but
               | not the need for constant use, and more isloation would
               | reduce both, but it would still likely take several kw
               | constantly on days like that.
        
             | bozhark wrote:
             | Had to delete my one line response because you embodied
             | everything I meant. Cheers
        
             | epistasis wrote:
             | > There's not enough time to do so with batteries alone.
             | 
             | I suggest you run the numbers on this, because I think you
             | have them exactly reversed.
             | 
             | We are increasing battery production capacity 10x every
             | five years. This could be accelerated if there was
             | government investment as is done for every single nuclear
             | reactor. At current rates we expect to produce
             | 20-30TWh/year of lithium ion batteries, not including other
             | chemistries that could be used for stationary storage but
             | not mobile applications.
             | 
             | Currently in the US we are only building 2GW of reactors,
             | but we have ~100GW of reactors quickly reaching retirement
             | age. Even if we scale our current nuclear construction
             | capacity 10x every five years like we do for batteries, and
             | add in the 10 year construction time, we are going to see a
             | big decrease in nuclear before we see an increase.
        
               | epistasis wrote:
               | Argh, there's a typo there, we expect 20-30TWh/year in
               | 2031.
        
             | samstave wrote:
             | I have long thought there should be a global nuclear
             | consortium and have that organization build and run and
             | manage and secure all the nuclear power plants in the
             | world.
             | 
             | Earth/Humanity needs electricity FOREVER. Its bizarre that
             | we cant come together over this expressly universal need.
             | This and water.
        
             | freemint wrote:
             | > But fission is the only answer we currently have for
             | baseline power that doesn't involve burning things.
             | 
             | Patently false. The sun shines 24/7. This (besides power
             | satellites) offers other possibilities like a world wide
             | connected grid. Even if you are a proponent of nuclear you
             | will also need that, as i assume you do not want to put
             | reactors in every country.
        
           | [deleted]
        
           | belorn wrote:
           | > So why waste more money on an obsolete technology rather
           | than use it for solving the remaining issues with renewables
           | like energy storage? Invest that money in battery technology
           | and everything that comes with that.
           | 
           | A few years ago Sweden did a study on green hydrogen, the
           | energy storage that Germany and many other countries seem to
           | view as the best bet as a storage for places where solar +
           | daily discharging batteries won't work. The cost was then
           | around 10-20 times more expensive than nuclear. Those costs
           | has gone down a bit since then, but it is still several times
           | more expensive than nuclear.
           | 
           | Sweden and Germany are still very much in favor of green
           | hydrogen, and there are on-going experiment to use it for
           | industries that need hydrogen itself (rather than burning it
           | for energy), but they are no investments for a grid storage.
           | If nuclear is not economically viable, a technology that is
           | several time more expensive is not something they are just
           | going to throw money at. Those money are currently going
           | towards fossil fuels, since that is cheaper than nuclear.
           | 
           | If however we would ban fossil fuel, especially cheap fossil
           | fuel from Russia, the economics might change. There is also
           | always the hope that politicians investment into fossil fuels
           | today will give green hydrogen enough time to become
           | economical viable for the energy sector.
        
             | pfdietz wrote:
             | China is selling electrolyzers for < $300/kw. Given that
             | renewable electricity's LCOE is a fraction of nuclear, I
             | don't see how hydrogen could be 10-20 times the cost of
             | nuclear. Were they doing something ridiculous like assuming
             | it's stored as liquid hydrogen?
             | 
             | Also, remember the big use of hydrogen on the grid would be
             | as a dispatchable backstop to cheap renewable sources, not
             | as something that's used 24/7. So most of the energy flow
             | would not be through hydrogen, it would be from the
             | renewables directly (or through batteries for short term
             | smoothing.)
        
               | belorn wrote:
               | There is a massive war and shortage of natural gas in
               | Europe so if there exist cheap electrolyzers + wind power
               | combinations that can solve that issue today then people
               | should rush to invest before next winter where prices are
               | predicted to sky rock. I recall that the study did say
               | that existing natural gas power plants could cheaply and
               | easily be converted to run on hydrogen. Hydrogen prices
               | has also gone up a lot since the war.
               | 
               | And it was 10-20 times a few years ago. Prices has gone
               | down a significant bit. If you get the prices to around
               | $1 per/kg (about 3-10x reduction from this year prices),
               | and we don't account for transportation, infrastructure
               | and physical storage, the price would start to look
               | really competitive to nuclear.
               | 
               | If you search online you will find plenty of predictions
               | that prices _might_ reach that magical $1 per /kg in say
               | 2030 or so, in which case that will be a great choice. As
               | a bonus it will make medical oxygen dirt cheap. At that
               | point all discussions about nuclear power will mostly be
               | made moot since hydrogen will be the factual best choice.
        
               | freemint wrote:
               | Hydrogen needs an complete overhaul to the pipeline
               | system. Burning natural gas for energy is a fraction of
               | it's use. More important are heating (where it can't be
               | transported too), steel making and chemistry (CoViD
               | vaccine ingredients are made using natural gas by BASF).
        
               | trashtester wrote:
               | This study from MIT found that the LCOE of fully
               | renewable energy production, backed by LI batteries would
               | be $3000/MWh vs $2400/MWh for hydrogen instead of
               | batteries for storage:
               | 
               | https://www.greencarcongress.com/2021/08/20210829-mitei.h
               | tml
               | 
               | Either alternative is about 15x to 50x more expensive
               | than Nuclear, though....
        
               | freemint wrote:
               | I thought the article we are replying to said
               | >>$4000+/MWh for nuclear not including financing.
        
               | trashtester wrote:
               | No, it is saying that it is currently about $6000000/MW
               | construction cost. Then you devide by number of hours of
               | operations to get the cost per MW/h. (Not adjusted for
               | interest rate.)
               | 
               | The LCOE of new nuclear plants have estimates ranging
               | from less than $30/MWh to around $150/MWh, while
               | estimates for the cost of plants built a few decades ago
               | end up at around $40-60/MWh, from the numbers I've seen.
        
               | pfdietz wrote:
               | That's grossly excessive. This website lets you look at
               | optimization vs. actual historical weather data and
               | reaches a much lower cost. BTW, you use both batteries
               | AND storage; their combination can be cheaper than either
               | alone. You also store the hydrogen underground rather
               | than above ground.
               | 
               | https://model.energy/
        
               | trashtester wrote:
               | I agree that a combination of batteries and hydrogen
               | would be a bit cheaper, but generally I would trust the
               | MIT study over the model above, that states directly that
               | it is a toy model.
               | 
               | It's fun to play with, though, so definitely upvoted.
               | 
               | Do you have an alternative peer reviewed study to support
               | the conclusions?
        
         | mellavora wrote:
         | Huh. I'd argue that they should be state-sponsored because that
         | is what the state is for-- investing in long-term
         | infrastructure where the value is widely diffused and thus hard
         | to capture via market mechanisms, i.e. investments which are
         | foundational to the more focused investments which business is
         | so well suited to maximize.
         | 
         | similar to things like interstate highways or internets or
         | public schooling. I'd add "healthcare" to the list, but some
         | countries haven't realized this yet.
        
           | joe_the_user wrote:
           | Indeed, US large-construction has reached a point of
           | fundamental/terminal corruption through the process of
           | private contracting and especially through the bid-based
           | system.
           | 
           | You can see this with the disastrous failure of public
           | transit construction and planning in the country and if
           | nuclear became an option, it seems very likely that the same
           | corrupt mouths that eat up transit spend would also eat
           | nuclear spending. Something for nuclear proponents to
           | consider.
        
             | belorn wrote:
             | One of the largest problem is the concept of cost-plus
             | contracts, where the construction doesn't have a fixed cost
             | from the beginning. Government could in theory avoid this
             | problem by doing the design first and then have companies
             | bid on the construction without a cost-plus contract. This
             | assumes that the regulations remain constant from start to
             | finish, as well as the contractual obligations.
        
       | balaji1 wrote:
       | Might be nitpicking here: Referring to the pie charts under this
       | section[1], I can't help but think images like this are
       | intentionally meant to mislead:
       | [1]https://constructionphysics.substack.com/p/why-are-
       | nuclear-p...
       | 
       | I understand author picked it from some other source, Dawson 2017
       | or whatever.
       | 
       | These pie charts are not like for like, when it includes the
       | renewables -- maybe the coal/gas/nuclear fine. So why put them
       | next to one another. I can't tell if the costs of renewables are
       | for entire "farms" (solar or wind farms). What about lifetime of
       | plants/farms, total value derived, etc.
        
         | zbrozek wrote:
         | What is your objection? The charts are all various fractional
         | contributions to LCOE, which already captures and normalizes
         | for things like plant life. You could argue that the size of
         | the pie should be scaled for the total LCOE, but that doesn't
         | really help with the comparison being made (e.g., the
         | relatively small fractional cost of fuel vs capital for nuclear
         | plants).
        
       | Ericson2314 wrote:
       | > but all else being equal (ie: assuming total production cost
       | stays constant), it's better to have a larger fraction of your
       | electricity costs be variable, so that if demand drops then
       | production cost drops as well.
       | 
       | This is how capitalism gives us scarcity when there is no reason
       | for scarcity. Infrastructure like trains and nuclear that
       | absolutely _swamps_ current demand, and whose costs cannot be
       | adjusted very much (even if one can run fewer trains, remove some
       | fuel rods) is _good_. But woe unto anyone that floods the market
       | utopia-style under capitalism --- you will just get paid nothing
       | and your investors will not repeat such a project again.
        
         | bgilroy26 wrote:
         | Cow's milk in the United States seems to be an exception to
         | this
        
         | skybrian wrote:
         | Another way to interpret that sentence is "it's better not to
         | spend too much on construction, and it can be better to spend
         | on fuel if it means spending less on construction."
         | 
         | Building things you don't need is waste of labor and resources,
         | regardless of the economic system you use or who paid for it.
         | This should be accounted for somehow.
        
           | Ericson2314 wrote:
           | What is needed changes over time.
           | 
           | It is better to spend more on capital than operational costs
           | if one actually wants the world to change not just ideal
           | along.
           | 
           | The best cost control is in large part doing lots of cookie-
           | cutter work, hence the focus on small modular reactors. It is
           | better to do those, and then ramp up the "small" part over
           | time.
           | 
           | Likewise, we should construct lots of rail simultaneously
           | with ramp up to improve those supply chains too.
           | 
           | But at no point do you _want_ to spend more on fuel in any
           | global optimal sense. That is just wasteful. It 's not like
           | we are _actually_ uncertain that we won 't need way more
           | electricity production on a societal scale.
        
             | skybrian wrote:
             | I agree that we are going to need more power generation as
             | electricity replaces other forms of energy.
             | 
             | But for any given level of electricity generation, doing it
             | in a cheaper way is still better than doing it in a more
             | expensive way. For one thing, it allows capacity to be
             | increased for the same cost.
        
         | LatteLazy wrote:
         | If your costs are all fixed, and demand falls, you have to put
         | prices up to stay alive. That's the issue with Nuclear: If it
         | costs $1bn a year for the plant, it costs that whether you
         | generate 1bn kWh (at $1 each) or 1 kWh (at $1Bn each).
         | 
         | What you call artificial shortage, others call "not making
         | something no one wants and then forcing them to pay for it"...
        
           | Ericson2314 wrote:
           | There is no non-depressing future where demand for
           | electricity doesn't go _way_ up as fossil fuel is phased out.
           | An uncertainty is a shit situation we should work to prevent,
           | not have a contingency plan for.
           | 
           | This is where the Keynesian "socialization of investment"
           | stuff comes in. Private markets get skiddish over change even
           | when our economies are fully capable of dealing with the
           | issues. Don't be held hostage to private capital playing
           | "nose goes" when the solution is obvious.
           | 
           | Abundance really does mean everything is too cheap to meter
           | on margin. There is no other definition. To get there we have
           | to make people more risk tolerant, and the only way to do
           | that is guaranteeing consumption.
        
             | LatteLazy wrote:
             | It's not so much that your points are wrong, as that they
             | fail to see a bigger picture.
             | 
             | When you say there is no no depressing future without
             | increased electrical demand, that's true. But who
             | guaranteed you a non depressing future?
             | 
             | And the same was true in 1990, but we've made almost no
             | progress to removing fossil fuels. So if you'd started
             | nuclear projects then, you'd be just coming online. And
             | have no new customers. And in the mean time the price of
             | gas and solar and wind would have collapsed in comparison.
             | And you couldn't afford to shut down when the price crashed
             | like they can. You're fixed costs would rapidly drive you
             | bankrupt.
             | 
             | That's the problem here.
             | 
             | The future isn't nice. And it's not predictable. Even over
             | relatively short periods.
             | 
             | So it's much better to plan 6m ahead and then do it again
             | 40 times, than to try and plan 20 years ahead in one go.
             | That makes it much much better to run small incremental
             | short-term projects like gas plants and renewables.
             | 
             | And if you do have to make some sort of medium term plan
             | then it needs to be very flexible. Like a gas plant that
             | can shut down for a year and avoid 90% of its costs.
             | 
             | Nuclear is long term, fixed cost. And that's terrible.
             | 
             | But go ahead, by some shares in a nuclear operator (or
             | someone else in the supply chain etc). Make your fortune
             | being right in 2045. Just don't sign up the rest of us via
             | the public finances please!
             | 
             | This has always been one of the issues with control
             | economies: people vastly over estimate their ability to
             | predict the future, they don't appreciate flexibility and
             | they don't manage risk. That's how the USSR ended up
             | producing enormous amounts of steel and no washing
             | machines: no one asked what people wanted, they just said
             | 20% more of the same compared to last year! China are doing
             | the same thing right now with electricity targets and no
             | financial services...
        
       | thraway11 wrote:
       | "For instance, in the 1980s several nuclear power plants in
       | Washington were canceled after the estimated construction costs
       | increased from $4.1 billion to over $24 billion."
       | 
       | That's not even 10x. I would say hitting the correct order of
       | magnitude is "on budget" for civil engineering projects.
       | 
       | Here is what out of budget looks like:
       | 
       | https://www.bloomberg.com/news/articles/2015-10-13/how-the-c...
        
       | [deleted]
        
       | mch82 wrote:
       | The cost of nuclear energy seems seriously understated.
       | 
       | Nuclear waste storage costs the US $6B per year and we still
       | don't have a long-term storage plan. We'll incur this cost for
       | hundreds or thousands of years.
       | https://earth.stanford.edu/news/steep-costs-nuclear-waste-us...
       | 
       | There's also a cost to secure the nuclear infrastructure.
       | https://medium.com/the-future-is-electric/nuclear-generation...
       | 
       | Can anyone find a cost per Megawatt Hour calculation for nuclear
       | that includes security and long-term waste storage? The
       | calculations I've found for the $96/MWh figure seem like they
       | include construction and operation costs, but exclude
       | decommissioning and waste management.
        
         | ncmncm wrote:
         | Waste management is by far the smallest of all the problems
         | with nukes.
         | 
         | Overwhelmingly the biggest problem is wholly legal corruption,
         | as cost and schedule are allowed to balloon without limit. For
         | everyone involved, the gravy train stops when a plant is
         | delivered, which no one actually involved wants ever to happen.
         | No one has offered any plausible suggestion for how to contain
         | corruption costs for nuke construction, at least in the US,
         | never mind who should apply such containment.
         | 
         | No such dynamic is apparent in solar and wind projects, where
         | expected cost is easy to estimate as a multiple of generating
         | units -- panels and turbines.
         | 
         | We may expect that SMRs will not find traction specifically
         | because they do not seem to offer the conduit for graft that
         | bespoke nuke construction guarantees.
        
         | Guvante wrote:
         | It is complicated: $6B is supposedly related to nuclear waste
         | from the weapons program which is likely hard to compare to
         | power production sites for a huge number of reasons.
         | 
         | Technically there is a $40B fund paid for from taxes on nuclear
         | production to deal with the long term storage problem by NIMBY
         | has prevented that from going from a concept into a reality. As
         | with many things "we need X but not here" makes getting things
         | done hard.
         | 
         | So from a planning standpoint the long term problem is solved
         | but in reality it isn't. How you boil that down into a cost I
         | couldn't begin to solve.
        
         | adastra22 wrote:
         | Nuclear waste is only a problem if you don't allow breeder
         | reactors. Otherwise it is a fuel resource and an asset, not a
         | liability.
        
           | devoutsalsa wrote:
           | Nuclear waste isn't a problem if you drill a hole that's deep
           | enough, encase the waste in a "dry cask", and drop it in the
           | hole. Kyle Hill did a great video on this...
           | 
           | "We solved nuclear waste decades ago" =>
           | https://youtu.be/4aUODXeAM-k
        
             | ncmncm wrote:
             | Or just drop it in the ocean, as US, Brits and Russians
             | have preferred in the past.
             | 
             | But this has nothing to do with construction cost.
        
         | com2kid wrote:
         | > Nuclear waste storage costs the US $6B per year and we still
         | don't have a long-term storage plan. We'll incur this cost for
         | hundreds or thousands of years.
         | 
         | New plants produce a lot less waste.
         | 
         | And the reason nuclear waste seems so expensive is because we
         | have externalized the cost of most other forms of power
         | generation. Coal and gas, well we just let people get sick or
         | die.
         | 
         | Natural gas, fracking has huge negative impacts on communities,
         | lots of illness, but again, we don't count that as part of the
         | "financial cost" of natural gas.
         | 
         | Solar, we have the panels made in some other country, we have
         | the raw materials mined in some other country, we import the
         | final product, now it is clean energy.
         | 
         | (I support solar, but there _is_ an environmental cost to solar
         | panels!)
         | 
         | Nuclear makes us confront the waste up front, so now we are all
         | freaked out about it.
         | 
         | As has been pointed out repeatedly, you could take a football
         | field, dig down a few stories, and put all the nuclear waste
         | needed for energy generation in there, and then have enough
         | left over to keep storing spent fuel from new high efficiency
         | reactors until sometime until well after everyone reading HN is
         | dead and gone.
         | 
         | It is a crap shoot if the cost of paying the lawyers to deal
         | with all the lawsuits over where to build the storage site will
         | end up costing more or less than the storage itself.
        
           | jka wrote:
           | > As has been pointed out repeatedly, you could take a
           | football field, dig down a few stories, and put all the
           | nuclear waste needed for energy generation in there
           | 
           | Here are mentions of this talking point on Hacker News:
           | 
           | https://hn.algolia.com/?dateRange=all&page=0&prefix=true&que.
           | ..
        
       | belorn wrote:
       | It is interesting that personal costs was such huge part of the
       | increase in construction. Construction seems like a field that
       | automation has yet to really start to create waves, but I recall
       | seeing news about small step forward like scaled up 3d printing
       | with cement. Modular construction is an other concept I have not
       | heard much for in the context of nuclear plants.
        
         | freemint wrote:
         | You can never 3d print pre stressed cement.
        
       ___________________________________________________________________
       (page generated 2022-06-09 23:00 UTC)